[obm-l] Re: [obm-l] Re: [obm-l] Re: [obm-l] Re: [obm-l] Ajuda em combinatória! Probleminha fuleiro!

2024-08-07 Por tôpico Anderson Torres
Em qua., 7 de ago. de 2024 às 10:24, Armando Staib
 escreveu:
>
> Em 1 diagonal eu fiz elas iguais  ou diferentes.
> Qdo sao iguais 8*7*7*1/4
> Qdo sao diferentes 8*7*6*6/4
> Total 602
>
>
> Em qua, 7 de ago de 2024 08:50, Prof. Douglas Oliveira 
>  escreveu:
>>
>> A diferença do meu para o seu foi no segundo caso, em que considerei apenas 
>> 2 rotações.
>>
>> Em qua., 7 de ago. de 2024, 08:01, Marcelo Gonda Stangler 
>>  escreveu:
>>>
>>> Dúvida o problema em diagonais!

E se fosse um pentágono?

>>>
>>> Os casos em que a diagonal tem a mesma cor, e tem cores diferentes, são 
>>> casos disjuntos que totalizam os casos totais, e caso ambas diagonais sejam 
>>> iguais (dentro de seu par), só podemos ter 2 rotações, e se não sempre 
>>> poderemos ter 4 rotações. Segue o desenvolvimento
>>>
>>> 2 diagonais iguais:
>>>
>>> 8*7 colorações
>>>
>>> 2 rotações
>>>
>>> 28 no total
>>>
>>> 1 diagonal igual:
>>>
>>> 8*7*6 colorações
>>>
>>> 4 rotações
>>>
>>> 84 no total
>>>
>>> 0 diagonais iguais:
>>>
>>> 8*7*6*5 colorações
>>>
>>> 4 rotações
>>>
>>> 420 no total
>>>
>>>
>>> Total: 532
>>>
>>>
>>>
>>> Em ter., 6 de ago. de 2024 6:35 PM, Prof. Douglas Oliveira 
>>>  escreveu:

 Olá amigos, estou bem curioso com o seguinte probleminha que encontrei na 
 lista do POTI do Carlos Shine de combinatória, onde não sei se esqueci 
 algum caso e encontrei 616 (acho), a resposta do Shine é 1044 e coloquei 
 no chat gpt (rs) e ele falou a respeito de um tal de Burnside e encontrou 
 903. kk

 Preciso de uma ajudinha!

 Eis o problema:

 Exemplo 7. Mariana tem tinta guache de 8 cores diferentes e quer pintar os 
 quatro quadradinhos unitários de um quadrado de lado 2 de modo que casas 
 que têm um lado comum tenham cores diferentes. De quantas maneiras ela 
 pode fazer isso? (Duas colorações são iguais se uma pode ser obtida a 
 partir de outra através de uma rotação.)


 Att:

 Douglas Oliveira

 --
 Esta mensagem foi verificada pelo sistema de antivírus e
 acredita-se estar livre de perigo.
>>>
>>>
>>> --
>>> Esta mensagem foi verificada pelo sistema de antivírus e
>>> acredita-se estar livre de perigo.
>>
>>
>> --
>> Esta mensagem foi verificada pelo sistema de antivírus e
>> acredita-se estar livre de perigo.
>
>
> --
> Esta mensagem foi verificada pelo sistema de antivírus e
> acredita-se estar livre de perigo.

-- 
Esta mensagem foi verificada pelo sistema de antiv�rus e
 acredita-se estar livre de perigo.


=
Instru��es para entrar na lista, sair da lista e usar a lista em
http://www.mat.puc-rio.br/~obmlistas/obm-l.html
=


Re: [obm-l] IMO - Geometria

2024-07-22 Por tôpico Anderson Torres
Em seg., 22 de jul. de 2024 às 20:39, Gilberto Azevedo
 escreveu:
>
> Qual o problema mais difícil de geometria da história da IMO ?

Eu acho que a IMO da Índia rendeu o problema mais difícil de geometria.

> Sei que isso é muito relativo, mas em números, qual o problema de geometria 
> que teve menos pessoas com 7 pontos ?
> Alguém tem essa informação? Dissecar no site é uma missão rsrsrs

Eu faria um shell script para isso, mas não estou com tanta paciência hoje.

>
> --
> Esta mensagem foi verificada pelo sistema de antivírus e
> acredita-se estar livre de perigo.

-- 
Esta mensagem foi verificada pelo sistema de antiv�rus e
 acredita-se estar livre de perigo.


=
Instru��es para entrar na lista, sair da lista e usar a lista em
http://www.mat.puc-rio.br/~obmlistas/obm-l.html
=


Re: [obm-l] Encontrar geometricamente os focos de uma elipse

2024-06-07 Por tôpico Anderson Torres
Em sex., 7 de jun. de 2024 08:30, Marcelo Gomes 
escreveu:

> Olá a todos, bom dia.
>
> Por favor, alguém poderia me informar se seria possível realizar a
> construção geométrica para encontrar *os focos* de uma elipse somente com
> as informações abaixo?
>
> A elipse neste caso é a elipse de Steiner, que apresenta área mínima e
> está inscrita em um triângulo ABC.
>
> Dados:
>
>- Os 6 pontos por onde o traçado da elipse passa (3 deles são os
>pontos médios de cada lado do triângulo e os outros 3 são reflexões destes
>pontos médios em relação ao Baricentro do triângulo ABC).
>- Obs.: embora sejam dados os 6 pontos a elipse não está traçada. Só
>sabemos que ela passa por estes 6 pontos.
>- O centro da elipse, que é o Baricentro do triângulo ABC.
>- As duas retas perpendiculares que contém os eixos maior e menor da
>elipse (somente são dadas as retas sem quaisquer valores).
>
>
Isso tudo é o mesmo problema?
Ou são três problemas diferentes?

Se alguém puder auxiliar, agradeço muito.
>
> Obrigado, Marcelo.
>
> --
> Esta mensagem foi verificada pelo sistema de antivírus e
> acredita-se estar livre de perigo.

-- 
Esta mensagem foi verificada pelo sistema de antiv�rus e
 acredita-se estar livre de perigo.



[obm-l] Re: [obm-l] Re: [obm-l] Re: [obm-l] Re: [obm-l] Combinatória

2024-03-16 Por tôpico Anderson Torres
Em qua., 13 de mar. de 2024 às 13:07, Claudio Buffara
 escreveu:
>
> Mas este caso tem 7 pessoas. E o enunciado fala em 3 A e 3 C.
>
> On Wed, Mar 13, 2024 at 9:28 AM Pedro Júnior  
> wrote:
>>
>> Eu pensei sim, mas e os casos do tipo ACCACAC. Esse caso não entra na conta 
>> 6! - 2* 3!* 3!.
>>
>> Em qua., 13 de mar. de 2024 às 09:09, Claudio Buffara 
>>  escreveu:
>>>
>>> Pense no oposto: de quantas maneiras as crianças e adultos podem se sentar 
>>> separados uns dos outros.
>>>
>>> On Wed, Mar 13, 2024 at 8:39 AM Pedro Júnior  
>>> wrote:

 Olá pessoal, bom dia.
 Alguém poderia me ajudar nesse problema?

 Seis poltronas enfileiradas em um cinema e entram 3 adultos e 3 crianças. 
 De quantas maneiras podem sentar-se 2 crianças juntas e dois adultos 
 juntos?


Eu pensei numa maneira muito feia: enumeração na raça!

AAA CCC +

AACACC +
AACCAC +
AACCCA +

ACAACC +
ACACAC -
ACACCA -

CAAACC +
CAACAC -
CAACCC +

ACCAAC +
ACCACA -
ACCCAA +

CACAAC -
CACACA -
CACCAA +

CCAAAC +
CCAACA +
CCACAA +

CCC AAA +

20 possibilidades. Apenas 14 servem.
Cada uma nos permite permutar as crianças seis vezes, os adultos outras seis.
6x6x14, que dá meh.


 Desde já fico grato!

 --
 Esta mensagem foi verificada pelo sistema de antivírus e
 acredita-se estar livre de perigo.
>>>
>>>
>>> --
>>> Esta mensagem foi verificada pelo sistema de antivírus e
>>> acredita-se estar livre de perigo.
>>
>>
>>
>> --
>>
>> Pedro Jerônimo S. de O. Júnior
>>
>> Professor de Matemática
>>
>> Geo João Pessoa – PB
>>
>>
>> --
>> Esta mensagem foi verificada pelo sistema de antivírus e
>> acredita-se estar livre de perigo.
>
>
> --
> Esta mensagem foi verificada pelo sistema de antivírus e
> acredita-se estar livre de perigo.

-- 
Esta mensagem foi verificada pelo sistema de antiv�rus e
 acredita-se estar livre de perigo.


=
Instru��es para entrar na lista, sair da lista e usar a lista em
http://www.mat.puc-rio.br/~obmlistas/obm-l.html
=


Re: [obm-l] Divisibilidade, pedido de esclarecimento

2024-03-04 Por tôpico Anderson Torres
Em seg., 4 de mar. de 2024 às 09:53, Pedro José  escreveu:
>
> Bom dia!
> Mas provar que ocorrendo as duas está certo, não é o que foi pedido.

Não foi isso que ele fez. Ele demonstrou que ambas as expressões são
equivalentes a r==7s (mod17).
Portanto, ambas são equivalentes entre si.

> Pode ser que ocorrendo as duas esteja OK e também que haja pelo menos um 
> caso, que dá certo para a primeira assertiva e não ocorre para a segunda ou 
> pode ter pelo menos um caso que ocorra para a segunda e não ocorra para 
> primeira, já é suficiente para furar.
> O certo é:
> supor (i) e mostrar que ocorre(ii) e depois provar a volta, supor (ii) e 
> mostrar que ocorre (i).

Essa é uma das maneiras de se demonstrar equivalências, não a única.
A bem da verdade, você simplesmente reverteu a ida para provar a volta
- bastava mostrar que cada implicação era reversível para assim
economizar duas linhas.

> (i) 9r + 5s | 17. 17s + 17r | 17 (iii) logo 4*(i)-2(iii) ==>  2r - 14s | 17 
> (iv).
> Como 17s! 17 (v); (1v)+ (v) ==> 2r+3s | 17. Provada a ida. 17
> 2r +3s |17 (ii) . Mas 17r + 17 s | 17 (iii). (iii)- 4*(i) ==> 9r +5s | 17 
> Provada a volta.
> logo 9r + 5s | 17 <=> 2r+ 3s | 17 C.Q.D.
>
>
> Cordialmente,
> PJMS
>
> Em sáb., 2 de mar. de 2024 às 12:37, Marcone Borges 
>  escreveu:
>>
>> Sendo r e s inteiros, mostre que 9r +5s divide 17 se, e somente se, 2r + 3s 
>> divide 17.
>> De 9r + 5s ==0(mod 17), assim como de 2r + 3s ==0(mod17), segue que
>> r==7s (mod17). Daí sai a resposta.
>> Ou podemos mostrar o que foi pedido usando 9r + 5r +4(2r +3s) = 17(r + s)
>> Mas, do ponto de vista de quem elaborou a questão, por que vincular essas 
>> expressões ao fato de que quando uma for um múltiplo de 17 a outra também 
>> será?
>>
>> --
>> Esta mensagem foi verificada pelo sistema de antivírus e
>> acredita-se estar livre de perigo.
>
>
> --
> Esta mensagem foi verificada pelo sistema de antivírus e
> acredita-se estar livre de perigo.

-- 
Esta mensagem foi verificada pelo sistema de antiv�rus e
 acredita-se estar livre de perigo.


=
Instru��es para entrar na lista, sair da lista e usar a lista em
http://www.mat.puc-rio.br/~obmlistas/obm-l.html
=


Re: [obm-l] Divisibilidade, pedido de esclarecimento

2024-03-04 Por tôpico Anderson Torres
Em sáb., 2 de mar. de 2024 às 15:28, Claudio Buffara
 escreveu:
>
> Isso só perguntando pra quem elaborou a questão.
> Mas a ideia pode ter surgido quando, ao manipular expressões desse tipo, a 
> pessoa notou que:
> 9r + 5r +4(2r +3s) = 17(r + s)
> e isso a fez pensar no enunciado.

Eu me lembro de ter visto expressões semelhantes com outros módulos
(primos, por que será?) faz muito tempo.
Para mim o mais interessante é descobrir equivalências.
Por exemplo, se Ax+By é múltiplo de 17, quem seria C tal que x-Cy é
múltiplo de 7? Isso é basicamente uma classe de equivalência.

Na verdade daria para fazer o contrário:
se C não é múltiplo de 17, então Kx+y é múltiplo de 17 se e somente se
(CK mod 17)x+(C mod 17)y também for.
Daí é só reduzir CK e C módulo 17.

Com isso dá para gerar problemas interessantes:

- Se x+10y é múltiplo de 17, então 9x+90y, ou 9x+5y, são múltiplos de
y (e vice-versa)
- Se x+10y é múltiplo de 17, então 2x+20y, ou 2x+3y, são múltiplos de
y (e vice-versa)

Logo,
- Se 9x+5y é múltiplo de 17, então 2x+3y é múltiplo de y (e vice-versa).

>
>
> On Sat, Mar 2, 2024 at 12:37 PM Marcone Borges  
> wrote:
>>
>> Sendo r e s inteiros, mostre que 9r +5s divide 17 se, e somente se, 2r + 3s 
>> divide 17.
>> De 9r + 5s ==0(mod 17), assim como de 2r + 3s ==0(mod17), segue que
>> r==7s (mod17). Daí sai a resposta.
>> Ou podemos mostrar o que foi pedido usando 9r + 5r +4(2r +3s) = 17(r + s)
>> Mas, do ponto de vista de quem elaborou a questão, por que vincular essas 
>> expressões ao fato de que quando uma for um múltiplo de 17 a outra também 
>> será?
>>
>> --
>> Esta mensagem foi verificada pelo sistema de antivírus e
>> acredita-se estar livre de perigo.
>
>
> --
> Esta mensagem foi verificada pelo sistema de antivírus e
> acredita-se estar livre de perigo.

-- 
Esta mensagem foi verificada pelo sistema de antiv�rus e
 acredita-se estar livre de perigo.


=
Instru��es para entrar na lista, sair da lista e usar a lista em
http://www.mat.puc-rio.br/~obmlistas/obm-l.html
=


[obm-l] Re: [obm-l] Re: Const. de triângulo

2024-01-16 Por tôpico Anderson Torres
Em ter, 16 de jan de 2024 12:23, Claudio Buffara 
escreveu:

> "Há vários problemas de CT com duas soluções."
>
> Claro!...   Fora o óbvio , com infinitas soluções (todas
> semelhantes entre si...) tem o  se, por exemplo, A for agudo e a < b
> < a/sen(A).
>
> O Geogebra certamente é uma tremenda ferramenta.
> Mas quantos professores sabem usá-lo adequadamente?
>


1. Quantos professores têm contato? Mesmo fatorando o acesso à internet,
ainda creio serem poucos

2. Seria interessante se fosse adotado, é bem lúdico!


> []s,
> Claudio.
>
>
> On Mon, Jan 15, 2024 at 7:53 PM Luís Lopes  wrote:
>
>> Oi Claudio,
>>
>> Eu acho que para os problemas no contexto que estamos falando a álgebra
>> pode decidir. Como o 17-gon. É construtível mas talvez a construção em si
>> poderia não ser conhecida. Os problemas dados 3 pontos da lista do Wernick
>> também precisaram de pesquisas para se decidir. Mas não sei muito sobre o
>> assunto.
>>
>> Há vários problemas de CT com duas soluções.
>>
>> O problema do quadrilátero é muito legal e também muito difícil acho que
>> para qualquer um. Há soluções (não sei se são fundamentalmente diferentes)
>> no livro do Virgílio, Court e do FG-M.
>>
>> Mas, pra mim, a principal função destes problemas de construção e’
>> pedagógica.
>>
>> É isso aí. Muita criatividade. E o Geogebra pode ajudar muito.
>>
>> Abs,
>> Luís
>>
>>
>>
>> On Jan 14, 2024, at 11:21 AM, Claudio Buffara 
>> wrote:
>>
>> Não tenho dúvidas de que o nível de dificuldade destes problemas varia de
>> “trivial” até “extremamente difícil”. Talvez até existam problemas em
>> aberto - ninguém acha uma solução e nem consegue provar que não existe
>> solução.
>>
>> O problem dos dados e’ interessante: existem triplas de dados que
>> resultam em dois ou mais triângulos não congruentes? Os casos clássicos de
>> congruência sugerem que a resposta é não. Mas talvez alguns tipos de dado
>> sejam mais “fracos” e não determinem totalmente o triângulo.
>>
>> Saindo dos triângulos, um legal e não muito fácil (pra mim…) é construir
>> um quadrilátero inscritível dados os comprimentos dos lados.
>>
>> Mas, pra mim, a principal função destes problemas de construção e’
>> pedagógica. Inseridos num curso de geometria, eles são uma variante
>> interessante de problemas métricos (a enorme maioria dos problemas vistos
>> na escola) nos quais os estudantes precisam usar a criatividade pra aplicar
>> propriedades básicas de figuras geométricas simples mas de um jeito
>> diferente, com muito mais necessidade de visualização.
>>
>> []s,
>> Claudio
>>
>> Em dom., 14 de jan. de 2024 às 11:41, Luís Lopes 
>> escreveu:
>>
>>> Oi Claudio,
>>>
>>> Mando pra vc com CC pra lista pra fazer mais um teste e ver se a lista
>>> recebe. Reply não funciona.
>>>
>>> Outra maneira seria usando o triângulo AMaMb. Esse problema é simples.
>>> Mais interessantes são (d_a; e_a bissetrizes interna e externa)  e
>>>  e os primos esquecidos <,e_a>.
>>>
>>> Problemas com e_a não são muito vistos. Como aquele que apareceu no
>>> WhatsApp do Madeira: construir o triângulo retângulo dados D_b, D_c e X,
>>> ponto do incírculo na reta BC. Não considerei com E_b , E_c, a gente acaba
>>> esquecendo. Nem sei como seria. Ou até com X_a, ponto do
>>> A-exincírculo. A lista é enorme.
>>>
>>> Considere agora . Tirei o < _a>. Bem fácil. E como dados dois qq
>>> entre  o terceiro fica determinado (sem falar em
>>> B-C), então  e  também são fáceis. E  cai na
>>> categoria  e .
>>>
>>> O que pode ser um desafio é a discussão sobre os dados nos problemas
>>> . Todos eles têm somente uma solução (considerando triângulos não
>>> congruentes, a segunda solução no , m>=h> não conta). No  os
>>> dados têm que satisfazer d sin(A/2) < h <= d. Para  não sei como
>>> determinar.
>>>
>>> Abs,
>>> Luís
>>>
>>>
>>> On Jan 14, 2024, at 7:48 AM, Claudio Buffara 
>>> wrote:
>>>
>>> 
>>>
>>> Trace AM com comprimento m_a.
>>> Trace a circunferência com diâmetro AM.
>>> Trace AP com comprimento h_a e P na circunferência.
>>>
>>> * M será o ponto médio de BC e P o pé da altura relativa a A.
>>>
>>> Prolonga AM até MA', com AM = MA'.
>>>
>>> * AA' será a diagonal do paralelogramo ABA'C, cujas diagonais se
>>> bissectam em M.
>>>
>>> Traça arco capaz de 180-A sobre AA'.
>>>
>>> * Já que, num paralelogramo, ângulos consecutivos são suplementares.
>>>
>>> Chame de B o ponto de intersecção deste arco capaz com a reta PM.
>>> Marque C na reta PM tal que B-M-C e MC = MB.
>>> E acabou.
>>>
>>> Há outra solução marcando P na outra semicircunferência de diâmetro AM
>>> (a menos que h_a = m_a).
>>>
>>> []s,
>>> Claudio.
>>>
>>>
>>> On Sun, Jan 14, 2024 at 12:58 AM Luís Lopes 
>>> wrote:
>>>
 Saudações, oi Anderson,

 Soluções usando fórmulas servem para mostrar que o triângulo é
 construtível e qual é sua forma e tamanho. Já ajuda naquela parte - suponha
 o problema resolvido. Mas a construção procurada deverá ser feita usando as
 propriedades da figura.

 Posso mandar no privad

[obm-l] Re: [obm-l] Const. de triângulo

2024-01-14 Por tôpico Anderson Torres
Em dom., 14 de jan. de 2024 às 00:58, Luís Lopes
 escreveu:
>
> Saudações, oi Anderson,
>
> Soluções usando fórmulas servem para mostrar que o triângulo é construtível e 
> qual é sua forma e tamanho.

Mostrar que é construtível, neste caso, implica mostrar a construção.
E ela é recheada de

> Já ajuda naquela parte - suponha o problema resolvido. Mas a construção 
> procurada deverá ser feita usando as propriedades da figura.
>
> Posso mandar no privado para quem se interessar as construções com as figuras 
> que um correspondente me enviou. Esse que tem h_c/b como dado é bem 
> interessante.
>
> Agora o problema  pode ser resolvido de 3 ou mais maneiras. Com 
> medianas é sempre bom pensar em simetrias e paralelogramos.

Acabei de resolver. A minha ideia foi a fórmula da mediana, que é
basicamente uma lei dos cossenos no triângulo (b,c,180-A).
Com isso, fica fácil construir o arco de circunferência que olha o
dobro da mediana de um ângulo 180-A.

Como disse antes, a solução full eu mando outro dia.

>
> Luís
>
>
> --
> Esta mensagem foi verificada pelo sistema de antivírus e
>  acredita-se estar livre de perigo.
>
>
> =
> Instru�ões para entrar na lista, sair da lista e usar a lista em
> http://www.mat.puc-rio.br/~obmlistas/obm-l.html
> =

-- 
Esta mensagem foi verificada pelo sistema de antiv�rus e
 acredita-se estar livre de perigo.


=
Instru��es para entrar na lista, sair da lista e usar a lista em
http://www.mat.puc-rio.br/~obmlistas/obm-l.html
=


Re: [obm-l] Const. de triangulo

2024-01-13 Por tôpico Anderson Torres
Em qui, 11 de jan de 2024 17:32, Anderson Torres <
torres.anderson...@gmail.com> escreveu:

>
>
> Em qui, 11 de jan de 2024 12:40, Luís Lopes 
> escreveu:
>
>> Vou mandar um texto bem carequinha.
>>
>> h_a,m_a,h_c:b
>>
>
> Esse não fiz ainda.
>

Quanto a esse aqui, o máximo que consegui foi:

h_c/b = ach_c/abc = a.2S/4SR = a/2R = sin(A).

Então sabemos o ângulo A, h_a e m_a.

Com h_a e m_a nós construímos o ponto A e a reta BC.
O problemão agora é onde meter esse ângulo A nisso tudo.
Tentei algumas ideias mas sem sucesso...

CLARO, eu posso ser altamente ignorante e mostrar fórmulas para a,b,c em
termos de A, h_a e m_a, mas isso seria muita sacanagem.



> b+c,h_a,h_b:h_c
>>
>
> Bem, ah_a=bh_b=ch_c = 2S onde S é a área de ABC.
> Assim, c/b = hb/hc.
>
> Conhecendo b+c e c/b, obtemos c e b (é uma construção fácil via
> paralelismo).
>
> Conhecendo b, c e h_a, é fácil obter o triângulo: marca o segmento AHa, a
> perpendicular por A, e depois os círculos de raios b e c por A.
>
> Mais tarde eu escrevo isso de forma algorítmica.
>
>
>
>
>> Luís
>>
>>
>> --
>> Esta mensagem foi verificada pelo sistema de antivírus e
>>  acredita-se estar livre de perigo.
>>
>>
>> =
>> Instruções para entrar na lista, sair da lista e usar a lista em
>> http://www.mat.puc-rio.br/~obmlistas/obm-l.html
>> =
>>
>

-- 
Esta mensagem foi verificada pelo sistema de antiv�rus e
 acredita-se estar livre de perigo.



Re: [obm-l] Const. de triangulo

2024-01-11 Por tôpico Anderson Torres
Em qui, 11 de jan de 2024 17:59, Marcelo Gonda Stangler <
marcelo.gonda.stang...@gmail.com> escreveu:

> Qual o objetivo disso?
>

Dadas certas informações, construir um triângulo com régua e compasso


> Em qui., 11 de jan. de 2024 5:41 PM, Anderson Torres <
> torres.anderson...@gmail.com> escreveu:
>

>>
>> Em qui, 11 de jan de 2024 12:40, Luís Lopes 
>> escreveu:
>>
>>> Vou mandar um texto bem carequinha.
>>>
>>> h_a,m_a,h_c:b
>>>
>>
>> Esse não fiz ainda.
>>
>> b+c,h_a,h_b:h_c
>>>
>>
>> Bem, ah_a=bh_b=ch_c = 2S onde S é a área de ABC.
>> Assim, c/b = hb/hc.
>>
>> Conhecendo b+c e c/b, obtemos c e b (é uma construção fácil via
>> paralelismo).
>>
>> Conhecendo b, c e h_a, é fácil obter o triângulo: marca o segmento AHa, a
>> perpendicular por A, e depois os círculos de raios b e c por A.
>>
>> Mais tarde eu escrevo isso de forma algorítmica.
>>
>>
>>
>>
>>> Luís
>>>
>>>
>>> --
>>> Esta mensagem foi verificada pelo sistema de antivírus e
>>>  acredita-se estar livre de perigo.
>>>
>>>
>>> =
>>> Instruções para entrar na lista, sair da lista e usar a lista em
>>> http://www.mat.puc-rio.br/~obmlistas/obm-l.html
>>> =
>>>
>>
>> --
>> Esta mensagem foi verificada pelo sistema de antivírus e
>> acredita-se estar livre de perigo.
>
>
> --
> Esta mensagem foi verificada pelo sistema de antivírus e
> acredita-se estar livre de perigo.

-- 
Esta mensagem foi verificada pelo sistema de antiv�rus e
 acredita-se estar livre de perigo.



Re: [obm-l] Const. de triangulo

2024-01-11 Por tôpico Anderson Torres
Em qui, 11 de jan de 2024 12:40, Luís Lopes 
escreveu:

> Vou mandar um texto bem carequinha.
>
> h_a,m_a,h_c:b
>

Esse não fiz ainda.

b+c,h_a,h_b:h_c
>

Bem, ah_a=bh_b=ch_c = 2S onde S é a área de ABC.
Assim, c/b = hb/hc.

Conhecendo b+c e c/b, obtemos c e b (é uma construção fácil via
paralelismo).

Conhecendo b, c e h_a, é fácil obter o triângulo: marca o segmento AHa, a
perpendicular por A, e depois os círculos de raios b e c por A.

Mais tarde eu escrevo isso de forma algorítmica.




> Luís
>
>
> --
> Esta mensagem foi verificada pelo sistema de antivírus e
>  acredita-se estar livre de perigo.
>
>
> =
> Instruções para entrar na lista, sair da lista e usar a lista em
> http://www.mat.puc-rio.br/~obmlistas/obm-l.html
> =
>

-- 
Esta mensagem foi verificada pelo sistema de antiv�rus e
 acredita-se estar livre de perigo.



Re: [obm-l] Const. de triangulo

2024-01-11 Por tôpico Anderson Torres
Em qui, 11 de jan de 2024 12:40, Luís Lopes 
escreveu:

> Vou mandar um texto bem carequinha.
>
> h_a,m_a,h_c:b
> b+c,h_a,h_b:h_c
>


Eu não entendi nada.


> Luís
>
>
> --
> Esta mensagem foi verificada pelo sistema de antivírus e
>  acredita-se estar livre de perigo.
>
>
> =
> Instruções para entrar na lista, sair da lista e usar a lista em
> http://www.mat.puc-rio.br/~obmlistas/obm-l.html
> =
>

-- 
Esta mensagem foi verificada pelo sistema de antiv�rus e
 acredita-se estar livre de perigo.



Re: [obm-l] OBM 88 Problema 6.

2023-12-31 Por tôpico Anderson Torres
Em dom., 31 de dez. de 2023 às 00:56, Pedro José  escreveu:
>
> Boa noite!
> Cláudio, minha preocupação é com a solução em si da equação.
> O problema original pede que demonstre que k é um quadrado perfeito. Todas 
> soluções que vi são baseadas nas relações de Girad ou Vieta's fórmula como 
> chamam lá fora.
> Eu parti do conhecimento de que k tem de ser quadrado perfeito.
> Consegui provar que tirando as soluções triviais a=0 ou b=0 ou a=b=1 
> b>=raiz(k)
> Aí achei a primeira solução para a equação, sem perda de generalidade, 
> considerei a>b, a=b só ocorre para a=b=1 ou a=b=0. Lá fora acho que nem 
> consideram 0 natural. Seguem a risca como foi o postulado de Peano.

O enunciado original dizia INTEIRO POSITIVO, e não "natural".
Os proponentes da IMO têm uma certa noção dessas pequenas polêmicas,
então eles costumam ser bastante verbosos sobre se 0 é considerado ou
não parte das soluções.
Curiosidade: na França 0 é considerado positivo E negativo ao mesmo tempo.
https://mathfour.com/arithmetic/is-zero-positive-or-negative

> Então para cada k=w^2 com w>1
> Tem um conjunto com uma sequência infinita de soluções.
> Sk={si=(ai,bi,k): i natural e i>=1| s1=(w^3,w,w^2) e si+1=(ai*w^2-bi, ai, 
> w^2).
> Consigo provar que todos termos da sequência são soluções.
> Não consigo provar que se há uma solução (a*,b*, k*) então (a*,b*,k*) ou 
> (b*,a*, k*) pertence a sequência Sk para k=w^2.

Ué, você pode imitar a solução do problema original. Se (a*,b*,K) é
solução E não está na rota dourada, então é possível encontrar uma
solução menor fora da rota dourada também, e assim por diante até
chegar na solução minimal. Mas a solução minimal é justo a que inicia
a rota dourada, absurdo.

> Eu não acho a solução da equação, só do problema como foi pedido, mostrar que 
> k é um QP, sem no entanto achar todas as soluções
>
> Cordialmente,
> PJMS
>
> Em sex., 29 de dez. de 2023 09:18, Claudio Buffara 
>  escreveu:
>>
>> Dá um Google em "IMO 88".
>> Vai ter até vídeo com a solução deste problema.
>>
>> On Thu, Dec 28, 2023 at 4:35 PM Pedro José  wrote:
>>>
>>> Boa tarde!
>>> Com referência a esse problema criei uma conjectura, não consegui provar 
>>> com a pretensão de abranger todas as soluções da equação:
>>>
>>> (a^2+b^2)/(ab+1)= k, com a,b,k Naturais e a>1, b>1 e k>1 Fiz essa restrição 
>>> para retirar as soluções triviais.
>>> E SPG considerei a>b, já que a=b só ocorre para a=b=1, que está fora pela 
>>> restrição acima e por ser uma equação simétrica em relação à a e b.
>>> O problema era provar que k era um quadrado perfeito.
>>> Gostaria de saber se alguém teria conhecimento da resolução em si do 
>>> problema, i.e., quais ternos (a*,b*,k*) são solução da equação.
>>> Caso ninguém tenha resolvido a equação, ainda, gostaria como faço para dar 
>>> divulgação da minha conjectura, onde tenho a pretenção de ter encontrado 
>>> todas as soluções possíveis para a equação em epígrafe, no Universo dos 
>>> Naturais, com a restrição a>1, b>1 e K>1.
>>>
>>> Agradeço quem puder me orientar.
>>>
>>> Cordialmente,
>>> PJMS
>>>
>>> --
>>> Esta mensagem foi verificada pelo sistema de antivírus e
>>> acredita-se estar livre de perigo.
>>
>>
>> --
>> Esta mensagem foi verificada pelo sistema de antivírus e
>> acredita-se estar livre de perigo.
>
>
> --
> Esta mensagem foi verificada pelo sistema de antivírus e
> acredita-se estar livre de perigo.

-- 
Esta mensagem foi verificada pelo sistema de antiv�rus e
 acredita-se estar livre de perigo.


=
Instru��es para entrar na lista, sair da lista e usar a lista em
http://www.mat.puc-rio.br/~obmlistas/obm-l.html
=


Re: [obm-l] OBM 88 Problema 6.

2023-12-28 Por tôpico Anderson Torres
Em qui, 28 de dez de 2023 19:01, Pedro José  escreveu:

> E daí?
>

E daí e daí?


> Em qui., 28 de dez. de 2023 18:42, Anderson Torres <
> torres.anderson...@gmail.com> escreveu:
>
>> Isso não é da OBM mas da IMO
>>
>> Em qui, 28 de dez de 2023 16:35, Pedro José 
>> escreveu:
>>
>>> Boa tarde!
>>> Com referência a esse problema criei uma conjectura, não consegui provar
>>> com a pretensão de abranger todas as soluções da equação:
>>>
>>> (a^2+b^2)/(ab+1)= k, com a,b,k Naturais e a>1, b>1 e k>1 Fiz essa
>>> restrição para retirar as soluções triviais.
>>> E SPG considerei a>b, já que a=b só ocorre para a=b=1, que está fora
>>> pela restrição acima e por ser uma equação simétrica em relação à a e b.
>>> O problema era provar que k era um quadrado perfeito.
>>> Gostaria de saber se alguém teria conhecimento da resolução em si do
>>> problema, i.e., quais ternos (a*,b*,k*) são solução da equação.
>>>
>>
>> Sim, o próprio método de resolução por descenso provê um método de
>> listagem das soluções.
>>
>> Caso ninguém tenha resolvido a equação, ainda, gostaria como faço para
>>> dar divulgação da minha conjectura, onde tenho a pretenção de ter
>>> encontrado todas as soluções possíveis para a equação em epígrafe, no
>>> Universo dos Naturais, com a restrição a>1, b>1 e K>1.
>>>
>>> Agradeço quem puder me orientar.
>>>
>>> Cordialmente,
>>> PJMS
>>>
>>> --
>>> Esta mensagem foi verificada pelo sistema de antivírus e
>>> acredita-se estar livre de perigo.
>>
>>
>> --
>> Esta mensagem foi verificada pelo sistema de antivírus e
>> acredita-se estar livre de perigo.
>
>
> --
> Esta mensagem foi verificada pelo sistema de antivírus e
> acredita-se estar livre de perigo.

-- 
Esta mensagem foi verificada pelo sistema de antiv�rus e
 acredita-se estar livre de perigo.



Re: [obm-l] OBM 88 Problema 6.

2023-12-28 Por tôpico Anderson Torres
Isso não é da OBM mas da IMO

Em qui, 28 de dez de 2023 16:35, Pedro José  escreveu:

> Boa tarde!
> Com referência a esse problema criei uma conjectura, não consegui provar
> com a pretensão de abranger todas as soluções da equação:
>
> (a^2+b^2)/(ab+1)= k, com a,b,k Naturais e a>1, b>1 e k>1 Fiz essa
> restrição para retirar as soluções triviais.
> E SPG considerei a>b, já que a=b só ocorre para a=b=1, que está fora pela
> restrição acima e por ser uma equação simétrica em relação à a e b.
> O problema era provar que k era um quadrado perfeito.
> Gostaria de saber se alguém teria conhecimento da resolução em si do
> problema, i.e., quais ternos (a*,b*,k*) são solução da equação.
>

Sim, o próprio método de resolução por descenso provê um método de listagem
das soluções.

Caso ninguém tenha resolvido a equação, ainda, gostaria como faço para dar
> divulgação da minha conjectura, onde tenho a pretenção de ter encontrado
> todas as soluções possíveis para a equação em epígrafe, no Universo dos
> Naturais, com a restrição a>1, b>1 e K>1.
>
> Agradeço quem puder me orientar.
>
> Cordialmente,
> PJMS
>
> --
> Esta mensagem foi verificada pelo sistema de antivírus e
> acredita-se estar livre de perigo.

-- 
Esta mensagem foi verificada pelo sistema de antiv�rus e
 acredita-se estar livre de perigo.



Re: [obm-l] OBM 88 Problema 6.

2023-12-28 Por tôpico Anderson Torres
Em qui, 28 de dez de 2023 17:40, Bruno Bianchi Pagani <
brunobianchipag...@gmail.com> escreveu:

> Como que eu saio disso?
>

procure pelas instruções de unsubscribe.


> On Thu, Dec 28, 2023, 4:35 PM Pedro José  wrote:
>
>> Boa tarde!
>> Com referência a esse problema criei uma conjectura, não consegui provar
>> com a pretensão de abranger todas as soluções da equação:
>>
>> (a^2+b^2)/(ab+1)= k, com a,b,k Naturais e a>1, b>1 e k>1 Fiz essa
>> restrição para retirar as soluções triviais.
>> E SPG considerei a>b, já que a=b só ocorre para a=b=1, que está fora pela
>> restrição acima e por ser uma equação simétrica em relação à a e b.
>> O problema era provar que k era um quadrado perfeito.
>> Gostaria de saber se alguém teria conhecimento da resolução em si do
>> problema, i.e., quais ternos (a*,b*,k*) são solução da equação.
>> Caso ninguém tenha resolvido a equação, ainda, gostaria como faço para
>> dar divulgação da minha conjectura, onde tenho a pretenção de ter
>> encontrado todas as soluções possíveis para a equação em epígrafe, no
>> Universo dos Naturais, com a restrição a>1, b>1 e K>1.
>>
>> Agradeço quem puder me orientar.
>>
>> Cordialmente,
>> PJMS
>>
>> --
>> Esta mensagem foi verificada pelo sistema de antivírus e
>> acredita-se estar livre de perigo.
>
>
> --
> Esta mensagem foi verificada pelo sistema de antivírus e
> acredita-se estar livre de perigo.

-- 
Esta mensagem foi verificada pelo sistema de antiv�rus e
 acredita-se estar livre de perigo.



[obm-l] O que aconteceu com os backups do prof. Carlos Shine no site de treinamento IMO/IBERO?

2023-12-05 Por tôpico Anderson Torres
Olá, pessoas!

O site https://imoibero.blogspot.com/ mantém alguns arquivos de
treinamentos antigos da IMO e IBERO. Mas os links estão quebrados.

Alguém tem as cópias ou sabe como posso contatar o webmaster para reavê-las?

-- 
Esta mensagem foi verificada pelo sistema de antiv�rus e
 acredita-se estar livre de perigo.


=
Instru��es para entrar na lista, sair da lista e usar a lista em
http://www.mat.puc-rio.br/~obmlistas/obm-l.html
=


[obm-l] Re: [obm-l] Números primos

2023-10-05 Por tôpico Anderson Torres
Em qua, 4 de out de 2023 15:49, carlos h Souza 
escreveu:

> Boa tarde,
>
> Para fins didáticos é mais fácil encontrar os números primos em forma de
> fatoração numérica ou usar o Crivo de Eratóstenes ?
>


Fatoração, de longe.

Os primos são definidos precisamente como "os infatoráveis".

Já o crivo de Eratóstenes é um algoritmo de classificação em massa.

Pensa da seguinte forma: para verificar se um número N é primo, o que é
mais natural:
- tentar dividir em k partes iguais, para todos os k pequenos;
- escrever todos os números de 1 a N num papel e ir furando o papel de
acordo com uma regra mágica?



> Obrigados a todos.
>
> --
> Esta mensagem foi verificada pelo sistema de antivírus e
> acredita-se estar livre de perigo.

-- 
Esta mensagem foi verificada pelo sistema de antiv�rus e
 acredita-se estar livre de perigo.



Re: [obm-l] Conjuntos

2023-08-08 Por tôpico Anderson Torres
mande uma vez somente.

Em ter, 8 de ago de 2023 12:33, Jamil Silva 
escreveu:

> Quantos conjuntos de três números inteiros positivos menores ou iguais a
> 2023 contêm a média aritmética de seus elementos ?
>
> --
> Esta mensagem foi verificada pelo sistema de antivírus e
> acredita-se estar livre de perigo.
>

-- 
Esta mensagem foi verificada pelo sistema de antiv�rus e
 acredita-se estar livre de perigo.



Re: [obm-l] Contagem

2023-08-08 Por tôpico Anderson Torres
Em ter, 8 de ago de 2023 19:11, Anderson Torres <
torres.anderson...@gmail.com> escreveu:

>
>
> Em ter, 8 de ago de 2023 13:01, Jamil Silva  escreveu:
>
>> Quantos conjuntos de três números inteiros positivos menores ou iguais a
>> 2023 contêm a média aritmética de seus elementos ?
>>
>
> Bem, a ideia é escolher dois números diferentes e com a mesma paridade,
> pois eles determinarão a média - a qual será inteira e estará entre eles.
>
> Temos 1011 pares e 1012 ímpares, o que nos dá C(1011,2)+C(1012,2) = 1012^2.
>

Aqui errei a contagem, deveria ser 1011^2


> Agora tô curioso, como demonstrar que o resultado para 2n-1 é n^2 com
> bijeção?
>

2N+1, no caso aqui, dá N^2

Como associar trincas de PAs com casas de um tabuleiro de xadrez nXn?
>
> Talvez não seja algo muito bonito, porque para 2n o resultado é n^2-n.
>

Para 2N dá N^2-N, aqui tá certo.


>> --
>> Esta mensagem foi verificada pelo sistema de antivírus e
>> acredita-se estar livre de perigo.
>
>

-- 
Esta mensagem foi verificada pelo sistema de antiv�rus e
 acredita-se estar livre de perigo.



Re: [obm-l] Contagem

2023-08-08 Por tôpico Anderson Torres
Em ter, 8 de ago de 2023 13:01, Jamil Silva  escreveu:

> Quantos conjuntos de três números inteiros positivos menores ou iguais a
> 2023 contêm a média aritmética de seus elementos ?
>

Bem, a ideia é escolher dois números diferentes e com a mesma paridade,
pois eles determinarão a média - a qual será inteira e estará entre eles.

Temos 1011 pares e 1012 ímpares, o que nos dá C(1011,2)+C(1012,2) = 1012^2.

Agora tô curioso, como demonstrar que o resultado para 2n-1 é n^2 com
bijeção?
Como associar trincas de PAs com casas de um tabuleiro de xadrez nXn?

Talvez não seja algo muito bonito, porque para 2n o resultado é n^2-n.

>
> --
> Esta mensagem foi verificada pelo sistema de antivírus e
> acredita-se estar livre de perigo.

-- 
Esta mensagem foi verificada pelo sistema de antiv�rus e
 acredita-se estar livre de perigo.



Re: [obm-l] OBM 2022 Problema

2023-06-13 Por tôpico Anderson Torres
De onde é esse problema?

Em ter, 13 de jun de 2023 07:50, Prof. Douglas Oliveira <
profdouglaso.del...@gmail.com> escreveu:

> Determine o maior inteiro positivo k para o qual a afirmação é verdadeira:
> Dados k subconjuntos distintos do conjunto {1, 2, 3, ..., 2023}, cada um
> com 1011 elementos, é possível particionar os subconjuntos em duas coleções
> em  a forma que quaisquer dois subconjuntos na mesma coleção têm algum
> elemento em comum.
>
>
>
> Abraço do Douglas Oliveira.
>
> --
> Esta mensagem foi verificada pelo sistema de antivírus e
> acredita-se estar livre de perigo.

-- 
Esta mensagem foi verificada pelo sistema de antiv�rus e
 acredita-se estar livre de perigo.



Re: [obm-l] Diferencial de ordem superior

2023-06-11 Por tôpico Anderson Torres
Em qui, 8 de jun de 2023 09:03, Bob Roy  escreveu:

> Olá pessoal,
> A notação de leibniz para f´´(x) = d^2(f) / dx^2 é apenas uma notação ? ou
> podemos isolar os numeradores?
> Vejo em alguns livros colocando dx^2 como (dx)^2..
>

Sim, é apenas uma notação. Uma muito bem feita, mas apenas uma notação.



> Bob Roy
>
> --
> Esta mensagem foi verificada pelo sistema de antivírus e
> acredita-se estar livre de perigo.

-- 
Esta mensagem foi verificada pelo sistema de antiv�rus e
 acredita-se estar livre de perigo.



Re: [obm-l] Irracionalidade de pi

2023-05-26 Por tôpico Anderson Torres
Em sex, 26 de mai de 2023 18:25, Israel Meireles Chrisostomo <
israelmchrisost...@gmail.com> escreveu:

>
> eu quero uma audiência com um matemático profissional, eu acabei de provar
> a irracionalidade de pi. alguém com tempo para corrigir?
>


O que tem de especial nisso para desejar um matemático profissional?

-- 
> Israel Meireles Chrisostomo
>
> --
> Esta mensagem foi verificada pelo sistema de antivírus e
> acredita-se estar livre de perigo.

-- 
Esta mensagem foi verificada pelo sistema de antiv�rus e
 acredita-se estar livre de perigo.



[obm-l] Re: [obm-l] Uma recorrência diferente

2023-04-06 Por tôpico Anderson Torres
Em qua., 5 de abr. de 2023 às 23:40, Professor Vanderlei Nemitz <
vanderma...@gmail.com> escreveu:

> Oi, mestres!
>
> Estava resolvendo um problema de combinatória e obtive essa recorrência:
>
> *x(n) = x(n - 1) + (n - 1).x(n - 2), com x1 = 1 e x2 = 2*.
>
> Por exemplo, x3 = x2 + 2.x1 e x9 = x8 + 8.x7
>
> Como resolver quando os coeficientes não são todos constantes?
>

Nem sempre dá para esperar que uma recorrência seja "resolvível".

Você pode ir tabelando e graficando alguns valores para ver se surge alguma
coisa, e daí imaginar alguma fórmula, algo como "bem, isso aqui parece um
gráfico de um fatorial"...

E, na real, nem sempre é tão útil ter uma fórmula pronta. Um computador
pode muito bem calcular os termos dessa sequência, em um processo idêntico
ao que um ser humano faria só que mais rápido (e com menos chance de
errar). E o vestibular não exigiu calcular isso para n=100, mas apenas para
n=9.

Todavia, é uma boa fuçar. Pensei em algo como usar uma sequência auxiliar,
y(n)=x(n+1)/x(n). Isso nos daria uma outra recorrência, a saber,

y(n)=1+(n/y(n-1))

O que estranhamente gera uma espécie de fração contínua. Mas não consigo
imaginar algo muito melhor que isso.



>
> Apenas como curiosidade, o problema que originou a recorrência é:
> (IME - RJ) - Um professor dá um teste surpresa para uma turma de 9 alunos,
> e diz que o teste pode ser feito sozinho ou em grupos de 2 alunos. De
> quantas formas a turma pode ser organizar para fazer o teste? (Por exemplo,
> uma turma de 3 alunos pode ser organizar de 4 formas e uma turma de 4
> alunos pode se organizar de 10 formas)
>

>
>
> 
>  Não
> contém vírus.www.avast.com
> 
> <#m_-8406560445346393254_DAB4FAD8-2DD7-40BB-A1B8-4E2AA1F9FDF2>
>
> --
> Esta mensagem foi verificada pelo sistema de antivírus e
> acredita-se estar livre de perigo.

-- 
Esta mensagem foi verificada pelo sistema de antiv�rus e
 acredita-se estar livre de perigo.



Re: [obm-l] Cone Sul

2023-03-18 Por tôpico Anderson Torres
Em seg., 13 de mar. de 2023 às 10:42, Armando Staib
 escreveu:
>
> Rsse repositorio é PAGO  certo!?

Não.

>
> Em seg, 13 de mar de 2023 10:26, Ian Barquette  
> escreveu:
>>
>> O repositório da "Art of Problem Solving" é muito completo, porém as 
>> questões são em inglês
>>
>> Em seg., 13 de mar. de 2023 09:09, Pedro Júnior 
>>  escreveu:
>>>
>>> Olá pessoal, muito bom dia.
>>> Gostaria de saber se tem um site oficial da competição "Cone Sul de 
>>> Matemática"? Procurei o banco de provas pelo Google e não encontrei. Me 
>>> remete ao site da OBM e também não vi por lá.
>>>
>>> Desde já fico grato.
>>>
>>> --
>>> Esta mensagem foi verificada pelo sistema de antivírus e
>>> acredita-se estar livre de perigo.
>>
>>
>> --
>> Esta mensagem foi verificada pelo sistema de antivírus e
>> acredita-se estar livre de perigo.
>
>
> --
> Esta mensagem foi verificada pelo sistema de antivírus e
> acredita-se estar livre de perigo.

-- 
Esta mensagem foi verificada pelo sistema de antiv�rus e
 acredita-se estar livre de perigo.


=
Instru��es para entrar na lista, sair da lista e usar a lista em
http://www.mat.puc-rio.br/~obmlistas/obm-l.html
=


Re: [obm-l] Cone Sul

2023-03-13 Por tôpico Anderson Torres
Em seg, 13 de mar de 2023 09:09, Pedro Júnior 
escreveu:

> Olá pessoal, muito bom dia.
> Gostaria de saber se tem um site oficial da competição "Cone Sul de
> Matemática"? Procurei o banco de provas pelo Google e não encontrei. Me
> remete ao site da OBM e também não vi por lá.
>

Já tentou o Mathlinks? Se você só está à procura dos enunciados, lá tem.


> Desde já fico grato.
>
> --
> Esta mensagem foi verificada pelo sistema de antivírus e
> acredita-se estar livre de perigo.

-- 
Esta mensagem foi verificada pelo sistema de antiv�rus e
 acredita-se estar livre de perigo.



[obm-l] Re: [obm-l] Seria por distribuição binomial ou alguma recorrência

2023-02-28 Por tôpico Anderson Torres
Em ter, 28 de fev de 2023 11:52, Bianca Flores 
escreveu:

> Alguém poderia ajudar com essa questão: estou frustrada porque não consigo
> chegar ao gabarito E.
>
> Um estudante preenche, aleatoriamente e de forma independente cada uma das
> questões, um exame de múltipla escolha com 5 respostas possíveis (das quais
> apenas uma é correta) para cada uma de 25 questões. A probabilidade que ele
> acerte um número par de questões é dada por:
>
> (A)(1-(4/5)^25)/2
> (B)(1-(3/5)^25)/2
> (C)((3/5)^25)/2
> (D)(1+(4/5)^25)/2
> (E)(1+(3/5)^25)/2
>

Bem, a probabilidade de acertar k questões de n é C(n,k) (1/5)^k
(4/5)^(n-k).
Soma isso para todo k par (lá ele) e vê no que dá.


>
> Tento de todas as formar usar a distribuição binomial, alguma recorrência,
> mas sem sucesso.
> Bianca
>
> --
> Esta mensagem foi verificada pelo sistema de antivírus e
> acredita-se estar livre de perigo.
>

-- 
Esta mensagem foi verificada pelo sistema de antiv�rus e
 acredita-se estar livre de perigo.



[obm-l] Re: [obm-l] Re: [obm-l] cadeira de 3 pés

2023-01-23 Por tôpico Anderson Torres
Em seg, 23 de jan de 2023 11:54, Rogerio Ponce 
escreveu:

> Ola' Claudio!
> Eu diria que as duas explicações estão erradas, pois não se depende de
> ter apenas um plano definido pelas pontas dos pés, visto que uma
> cadeira de 4 pés pode, perfeitamente, ter as pontas dos 4 pés em
> apenas um plano, e, ainda assim, ela não é necessariamente estável.
>

Como não? A ideia de botar um calço é precisamente estabilizar o pé que não
encosta na superfície. E, exceto em feiras de ciências e circos, o chão
costuma ser plano.


> Um explicação menos ruim é que, numa cadeira de 3 pés, sempre podemos
> apoiar quaisquer 2 pés num piso (mesmo irregular), e, em torno do eixo
> definido pelos 2 pés já apoiados, podemos girar a cadeira até que o
> terceiro pé encontre o piso, de modo que a cadeira fique totalmente
> apoiada.
> Já numa cadeira de 4 pés, é comum que um dos pés fique sem contato com
> o chão, permitindo que a cadeira oscile em torno do eixo definido
> pelos 2 pés vizinhos ao pé sem contato.
>
> []'s
> Rogerio Ponce
>
> On Sun, Jan 22, 2023 at 11:23 PM Claudio Buffara
>  wrote:
> >
> > Achei na internet duas explicações distintas para a estabilidade de uma
> cadeira (ou mesa ou banco) de 3 pés.
> > Aqui estão:
> > https://www.somatematica.com.br/curiosidades/c98.php
> >
> http://colegiofarroupilha.com.br/site/qual-cadeira-e-mais-firme-a-que-tem-tres-ou-quatro-pes/
> >
> > Qual das duas é a explicação correta?
> > Ou nenhuma das duas? E, nesse caso, qual a explicação?
> >
> > []s,
> > Claudio.
> >
> > --
> > Esta mensagem foi verificada pelo sistema de antivírus e
> > acredita-se estar livre de perigo.
>
> --
> Esta mensagem foi verificada pelo sistema de antivírus e
>  acredita-se estar livre de perigo.
>
>
> =
> Instru�ões para entrar na lista, sair da lista e usar a lista em
> http://www.mat.puc-rio.br/~obmlistas/obm-l.html
> =
>

-- 
Esta mensagem foi verificada pelo sistema de antiv�rus e
 acredita-se estar livre de perigo.



[obm-l] Re: [obm-l] Re: [obm-l] Re: [obm-l] cadeira de 3 pés

2023-01-23 Por tôpico Anderson Torres
Em seg, 23 de jan de 2023 11:15, Claudio Buffara 
escreveu:

> Será que o argumento usando apenas o plano é suficiente?  Pois um banco de
> 3 pés também fica estável num piso irregular.
>

Mas nem toda cadeira de quatro pés fica estável em qualquer piso irregular.

A ideia subjacente ainda é a de "planidade". Os pés da cadeira estão na
intersecção entre o "plano dos pés" e o piso.
Mas o mesmo não se aplica às cadeiras quadrúpedes, pois nem sempre existe
um "plano dos pés".

Se bem que neste último caso é possível que os pés da mesa sejam coplanares
mas a superfície não o seja. Ou pior ainda, forçando um pouco na
continuidade, pode ser que qualquer cadeira de quatro pés se encaixe em
qualquer superfície não-plana.

Ou não. Se pegarmos por exemplo o mapa topográfico de uma planície com um
poço escavado e a distância entre dois pés da mesa sempre for maior que o
diâmetro do poço, não tem como encaixar uma mesa torta de quatro pés.

Estranho...

O resultado mais geral em que pensei foi o seguinte: dada qualquer
> superfície bi-dimensional contínua (por exemplo, que seja o gráfico de uma
> função contínua de RxR em R - uma suposição razoável se estamos tentando
> modelar um piso), você sempre consegue encostar nela as pontas dos 3 pés do
> banco, de modo que o banco fique "firme" ou sem folgas.
> Ou, mais formalmente, dado um triângulo ABC no espaço, existe uma
> isometria (do espaço) tal que as imagens de A, B e C por esta isometria
> estão em S.
>

Pensei algo do gênero, mas de maneira bem menos elaborada: dados um
triângulo ABC e dois pontos A',B' em uma superfície contínua que distam AB,
existe um ponto C´ tal que ABC = A'B'C'


> Enfim, esse talvez seja um problema mais de topologia do que de geometria.
> Pois, no fim das contas, "3 pontos não colineares determinam um único
> plano", assim como "2 pontos determinam uma única reta" são afirmações que
> têm um certo ar topológico, pelo menos pra mim.
>

> []s,
> Claudio.
>
> On Mon, Jan 23, 2023 at 7:02 AM Anderson Torres <
> torres.anderson...@gmail.com> wrote:
>
>>
>>
>> Em dom, 22 de jan de 2023 23:23, Claudio Buffara <
>> claudio.buff...@gmail.com> escreveu:
>>
>>> Achei na internet duas explicações distintas para a estabilidade de uma
>>> cadeira (ou mesa ou banco) de 3 pés.
>>> Aqui estão:
>>> https://www.somatematica.com.br/curiosidades/c98.php
>>>
>>> http://colegiofarroupilha.com.br/site/qual-cadeira-e-mais-firme-a-que-tem-tres-ou-quatro-pes/
>>>
>>
>> Nesse caso específico, a primeira me parece mais correta. Ou melhor, a
>> segunda tem falhas.
>>
>> A rigidez dos triângulos (TCC caso LLL de igualdade de triângulos) é
>> irrelevante para a questão da cadeira bamba. Em qualquer cadeira física e
>> palpável, as pernas são rígidas - portanto o polígono formado pelas
>> extremidades dessas pernas é rígido também.
>>
>> Mas um polígono rígido não é necessariamente um polígono bidimensional -
>> o que a cadeira bamba de 4 pernas exemplifica perfeitamente.
>>
>> Contate o webmaster da segunda página sugerindo correções!
>>
>>
>>> Qual das duas é a explicação correta?
>>> Ou nenhuma das duas? E, nesse caso, qual a explicação?
>>>
>>> []s,
>>> Claudio.
>>>
>>> --
>>> Esta mensagem foi verificada pelo sistema de antivírus e
>>> acredita-se estar livre de perigo.
>>
>>
>> --
>> Esta mensagem foi verificada pelo sistema de antivírus e
>> acredita-se estar livre de perigo.
>
>
> --
> Esta mensagem foi verificada pelo sistema de antivírus e
> acredita-se estar livre de perigo.

-- 
Esta mensagem foi verificada pelo sistema de antiv�rus e
 acredita-se estar livre de perigo.



[obm-l] Re: [obm-l] cadeira de 3 pés

2023-01-23 Por tôpico Anderson Torres
Em dom, 22 de jan de 2023 23:23, Claudio Buffara 
escreveu:

> Achei na internet duas explicações distintas para a estabilidade de uma
> cadeira (ou mesa ou banco) de 3 pés.
> Aqui estão:
> https://www.somatematica.com.br/curiosidades/c98.php
>
> http://colegiofarroupilha.com.br/site/qual-cadeira-e-mais-firme-a-que-tem-tres-ou-quatro-pes/
>

Nesse caso específico, a primeira me parece mais correta. Ou melhor, a
segunda tem falhas.

A rigidez dos triângulos (TCC caso LLL de igualdade de triângulos) é
irrelevante para a questão da cadeira bamba. Em qualquer cadeira física e
palpável, as pernas são rígidas - portanto o polígono formado pelas
extremidades dessas pernas é rígido também.

Mas um polígono rígido não é necessariamente um polígono bidimensional - o
que a cadeira bamba de 4 pernas exemplifica perfeitamente.

Contate o webmaster da segunda página sugerindo correções!


> Qual das duas é a explicação correta?
> Ou nenhuma das duas? E, nesse caso, qual a explicação?
>
> []s,
> Claudio.
>
> --
> Esta mensagem foi verificada pelo sistema de antivírus e
> acredita-se estar livre de perigo.

-- 
Esta mensagem foi verificada pelo sistema de antiv�rus e
 acredita-se estar livre de perigo.



Re: [obm-l] Irracionalidade de Pi

2023-01-22 Por tôpico Anderson Torres
Em sáb., 21 de jan. de 2023 às 13:27, Claudio Buffara
 escreveu:
>
> A demonstração tradicional da irracionalidade de Pi começa estabelecendo 
> algumas propriedades da função:
> x |--> x^n * (1-x)^n / n!
> no intervalo (0,1).
>
> Essa função me parece tirada da cartola, sem qualquer motivação prévia.
> Alguém sabe o que levou o autor da demonstração a usar esta função?

Bem, eu fiz uma rápida busca no Google por "motivated demonstration
irrationality pi" e encontrei isso:

"Discovering and Proving that π Is Irrational" por Timothy W. Jones

The American Mathematical Monthly
Vol. 117, No. 6 (June-July 2010), pp. 553-557 (5 pages)
https://doi.org/10.4169/000298910x492853

E também uns links extras:

https://mattbaker.blog/2015/03/15/a-motivated-and-simple-proof-that-pi-is-irrational/
https://math.stackexchange.com/questions/4051354/what-is-the-motivation-behind-the-steps-in-this-simple-proof-that-pi-is-irr
https://page.math.tu-berlin.de/~mdmv/archive/19/mdmv-19-zhou.pdf

Divirta-se :)

>
> []s,
> Claudio.
>
> --
> Esta mensagem foi verificada pelo sistema de antivírus e
> acredita-se estar livre de perigo.

-- 
Esta mensagem foi verificada pelo sistema de antiv�rus e
 acredita-se estar livre de perigo.


=
Instru��es para entrar na lista, sair da lista e usar a lista em
http://www.mat.puc-rio.br/~obmlistas/obm-l.html
=


[obm-l] Re: [obm-l] Mostrar que [n!]/e é sempre par

2022-12-15 Por tôpico Anderson Torres
Em sex, 16 de dez de 2022 00:53, Artur Costa Steiner <
artur.costa.stei...@gmail.com> escreveu:

> Problema interessante: Mostre que, para todo inteiro n >= 0,  [n!]/e é
> sempre par, sendo [x] o piso de x.
>

você quis dizer [n!/e] onde e é a base do log natural?

Bem, 1/e=e^(-1)=
(1/0!-1/1!)+(1/2!-1/3!)+(1/4!-1/5!)+(1/6!-1/7!)+... =
2/3! + 4/5! + 6/7! + 8/9! + ...

Dessa forma, [n!/e]=

[2n!/3! + 4n!/5! + 6n!/7! + 8n!/9! + ...]

Que, obviamente, redunda numa sema de pares após truncado (denominador
>n!),


> Abraços
>
> Artur
>
> --
> Esta mensagem foi verificada pelo sistema de antivírus e
> acredita-se estar livre de perigo.

-- 
Esta mensagem foi verificada pelo sistema de antiv�rus e
 acredita-se estar livre de perigo.



[obm-l] Re: [obm-l] Teoria dos números, trigonometria e racionalidade

2022-12-11 Por tôpico Anderson Torres
Em dom., 11 de dez. de 2022 às 10:32, Anderson Torres
 escreveu:
>
> Em sáb., 10 de dez. de 2022 às 22:08, marcone augusto araújo borges
>  escreveu:
> >
> > Seja p um número primo tal que p = = 3 (mod4) e @ um ângulo tal que tan@ é 
> > racional. Prove que tan((p+1)@) também é racional com numerador múltiplo de 
> > p
> > Desde já agradeço por algum esclarecimento ou solução.
>
> Bem, o que eu consigo pensar é em algo desse tipo.
>
> Sabemos que tan(m+n) = (tan(m) + tan(n))/(1-tan(m)* tan(n))
>
> Escrevamos tan(nX)=p(n)/q(n), onde p e q são polinômios em t=tan(X).
> Temos então a seguinte recorrência:
>
> p(1)=t; p(n+1)=p(n)+tq(n)
> q(1)=1; q(n+1)=-tp(n)+q(n)
>
> Jogando aqui e ali, temos
>
> p(1)=t; p(2)=2t; p(n+2)=2p(n+1)-(t^2+1)p(n)
> q(1)=1; q(2)=1-t^2; q(n+2)=2q(n+1)-(t^2+1)q(n)
>
> De cara, se nota que p sempre será múltiplo de p, e que q sempre deixa
> resto 1 módulo t, o que já dá uma pista do que procurar...
> Decerto, vai aparecer alguma coisa do tipo x^2+1, e com isso se usa o
> fato de p ser primo da forma 4k-1...
>

Acho que dá para melhorar. Suponha tan(nX)=A(n)/B(n). Assim,

A(n+1) =  B*A(n) + A*B(n)
B(n+1) = -A*A(n) + B*B(n)

E portanto

A(n+2) = 2B*A(n+1) - (A^2+B^2)*A(n), A(1)=A, A(2)=2AB
B(n+2) = 2B*B(n+1) - (A^2+B^2)*B(n), B(1)=B, B(2)=B^2-A^2

A ideia então seria demonstrar que A(p+1) é múltiplo de p para p primo
da forma 4k-1, e B(p+1) não é múltiplo de p para p primo da forma
4k-1.

Dessa forma, ao menos em princípio seria possível verificar a segunda
premissa, pois a primeira é óbvia.

> >
> > --
> > Esta mensagem foi verificada pelo sistema de antivírus e
> > acredita-se estar livre de perigo.

-- 
Esta mensagem foi verificada pelo sistema de antiv�rus e
 acredita-se estar livre de perigo.


=
Instru��es para entrar na lista, sair da lista e usar a lista em
http://www.mat.puc-rio.br/~obmlistas/obm-l.html
=


[obm-l] Re: [obm-l] OBM e Olímpiadas internacionais

2022-12-11 Por tôpico Anderson Torres
Em qua., 7 de dez. de 2022 às 03:39, Obindinachukwu Desire Yema
 escreveu:
>
> Bom dia a todos,
> Nesse ano eu despertei um interesse em matemática pura, pensando um pouco 
> decidi que iria tentar no próximo ano fazer a OBM nivel universitário. 
> Pesquisando no site da OBM, eu não achei nada relacionado com o conteúdo que 
> cai na prova.
> Eu queria perguntar para vocês como que me preparo para a prova, no sentido 
> de: conteúdo que devo saber.
> Desde já agradeço a atenção.

De fato tem pouca coisa além das provas. Então, te sugiro pegar
pesado: estude a PUTNAM e a IMC. Com isso você vai ter mais material.

>
> --
> Esta mensagem foi verificada pelo sistema de antivírus e
> acredita-se estar livre de perigo.

-- 
Esta mensagem foi verificada pelo sistema de antiv�rus e
 acredita-se estar livre de perigo.


=
Instru��es para entrar na lista, sair da lista e usar a lista em
http://www.mat.puc-rio.br/~obmlistas/obm-l.html
=


[obm-l] Re: [obm-l] Teoria dos números, trigonometria e racionalidade

2022-12-11 Por tôpico Anderson Torres
Em sáb., 10 de dez. de 2022 às 22:08, marcone augusto araújo borges
 escreveu:
>
> Seja p um número primo tal que p = = 3 (mod4) e @ um ângulo tal que tan@ é 
> racional. Prove que tan((p+1)@) também é racional com numerador múltiplo de p
> Desde já agradeço por algum esclarecimento ou solução.

Bem, o que eu consigo pensar é em algo desse tipo.

Sabemos que tan(m+n) = (tan(m) + tan(n))/(1-tan(m)* tan(n))

Escrevamos tan(nX)=p(n)/q(n), onde p e q são polinômios em t=tan(X).
Temos então a seguinte recorrência:

p(1)=t; p(n+1)=p(n)+tq(n)
q(1)=1; q(n+1)=-tp(n)+q(n)

Jogando aqui e ali, temos

p(1)=t; p(2)=2t; p(n+2)=2p(n+1)-(t^2+1)p(n)
q(1)=1; q(2)=1-t^2; q(n+2)=2q(n+1)-(t^2+1)q(n)

De cara, se nota que p sempre será múltiplo de p, e que q sempre deixa
resto 1 módulo t, o que já dá uma pista do que procurar...
Decerto, vai aparecer alguma coisa do tipo x^2+1, e com isso se usa o
fato de p ser primo da forma 4k-1...

>
> --
> Esta mensagem foi verificada pelo sistema de antivírus e
> acredita-se estar livre de perigo.

-- 
Esta mensagem foi verificada pelo sistema de antiv�rus e
 acredita-se estar livre de perigo.


=
Instru��es para entrar na lista, sair da lista e usar a lista em
http://www.mat.puc-rio.br/~obmlistas/obm-l.html
=


[obm-l] Re: [obm-l] Re: [obm-l] Re: [obm-l] Caracterização de Inteiros

2022-11-15 Por tôpico Anderson Torres
Em ter, 15 de nov de 2022 17:07, Pedro José  escreveu:

> Obrigado a você e ao Cláudio. Mas não sou criativo para inventar. Mas já
> vi que terei que fazer uma homotetia, para as classes de equivalência para
> representar só como um número e não como um par, creio eu.
>

Eu lembro de quando li o Guidorizzi formalizando os reais. Até hoje sinto
que entendo sem compreender, haha!

Por outro lado, números reais (irracionais, no caso) são bem menos
palpáveis que os outros. Dívidas e frações são fáceis de entender, afinal.


> Cordialmente,
> PJMS
>
> Em ter., 15 de nov. de 2022 às 16:00, Anderson Torres <
> torres.anderson...@gmail.com> escreveu:
>
>>
>>
>> Em ter, 15 de nov de 2022 14:33, Pedro José 
>> escreveu:
>>
>>> Boa tarde!
>>> Para os |Naturais, temos os postulados de Peano.
>>>
>>> Para os Inteiros há alguma formalização?
>>>
>>
>> invente uma!
>>
>> Pode ser por exemplo o conjunto de pares (p,q) tais que p-q é constante.
>>
>> ou melhor (p1,q1)=(p2,q2) se e só se p1+q2=p2+q1.
>>
>>
>>> Acho pobre dizer que é necessário ter outros números devido ao problema
>>> de fechamento nos naturais para a subtração que é fato e daí introduzir os
>>> simétricos que são inteiros e ainda não foram caracterizados.
>>>
>>> No meu antigo ginásio aprendi que os Reais era a união dos conjuntos
>>> disjuntos irracionais e racionais. Os racionais haviam sido bem definidos.
>>> Aí questionei e o que são irracionais? resposta: são os Reais que não são
>>> racionais, os que não podem ser escritos na forma p/q p e q inteiros e
>>> q<>0. Mas me deram um tombo. Definiram os |Reais com base nos irracionais e
>>> os irracionais com base nos |Reais. 3 +2i também não pode ser inscrito na
>>> forma p/q. Só mais tarde no científico, é que meu professor definiu
>>> irracional como um número que não podia ser escrito na forma p/q e cuja
>>> representação decimal tinha uma infinidade de algarismos, sem haver uma
>>> periodicidade.
>>> Na época foi o maior nó que tive com a matemática. O mestre demonstrou
>>> que os racionais eram densos, mas entre eles ainda cabiam os irracionais.
>>> Não satisfeito mostrou que os racionais eram enumeráveis e por absurdo
>>> mostrou que os |Reais não. Não satisfeito mostrou que a cardinalidade do
>>> intervalo [0,1] era maior que a dos |Naturais. Não conseguia conceber que
>>> havia um infinito maior que outro. Outra coisa que demorei a aceitar,mesmo
>>> vendo a bijeção, era que os inteiros e naturais tinham a mesma
>>> cardinalidade. Na minha cabeça, os inteiros têm todos os naturais ainda
>>> sobram os negativos, como é igual?
>>> Hoje, depois de velho, arrumei uma enteada, que muito me pergunta e
>>> estou enrolado. Para dar um ar de superioridade, questionei se conhecia os
>>> inteiros de Gaus, que 5 não era primo nos inteiros de Gaus. Estrepei-me, a
>>> danada foi pesquisar e me questiona sobre o que não tenho um domínio pleno.
>>> Em suma, como apresentei a ela os postulados de Peano para a
>>> caracterização dos Naturais, ela me cobra por algo semelhante para os
>>> Inteiros, e não sei responder.
>>> HELP! SOCORRO! AU SECOURS! AYUDA! AIUTO! HILFE!
>>> Cordialmente,
>>> PJMS
>>>
>>> --
>>> Esta mensagem foi verificada pelo sistema de antivírus e
>>> acredita-se estar livre de perigo.
>>
>>
>> --
>> Esta mensagem foi verificada pelo sistema de antivírus e
>> acredita-se estar livre de perigo.
>
>
> --
> Esta mensagem foi verificada pelo sistema de antivírus e
> acredita-se estar livre de perigo.

-- 
Esta mensagem foi verificada pelo sistema de antiv�rus e
 acredita-se estar livre de perigo.



[obm-l] Re: [obm-l] Caracterização de Inteiros

2022-11-15 Por tôpico Anderson Torres
Em ter, 15 de nov de 2022 14:33, Pedro José  escreveu:

> Boa tarde!
> Para os |Naturais, temos os postulados de Peano.
>
> Para os Inteiros há alguma formalização?
>

invente uma!

Pode ser por exemplo o conjunto de pares (p,q) tais que p-q é constante.

ou melhor (p1,q1)=(p2,q2) se e só se p1+q2=p2+q1.


> Acho pobre dizer que é necessário ter outros números devido ao problema de
> fechamento nos naturais para a subtração que é fato e daí introduzir os
> simétricos que são inteiros e ainda não foram caracterizados.
>
> No meu antigo ginásio aprendi que os Reais era a união dos conjuntos
> disjuntos irracionais e racionais. Os racionais haviam sido bem definidos.
> Aí questionei e o que são irracionais? resposta: são os Reais que não são
> racionais, os que não podem ser escritos na forma p/q p e q inteiros e
> q<>0. Mas me deram um tombo. Definiram os |Reais com base nos irracionais e
> os irracionais com base nos |Reais. 3 +2i também não pode ser inscrito na
> forma p/q. Só mais tarde no científico, é que meu professor definiu
> irracional como um número que não podia ser escrito na forma p/q e cuja
> representação decimal tinha uma infinidade de algarismos, sem haver uma
> periodicidade.
> Na época foi o maior nó que tive com a matemática. O mestre demonstrou que
> os racionais eram densos, mas entre eles ainda cabiam os irracionais. Não
> satisfeito mostrou que os racionais eram enumeráveis e por absurdo mostrou
> que os |Reais não. Não satisfeito mostrou que a cardinalidade do intervalo
> [0,1] era maior que a dos |Naturais. Não conseguia conceber que havia um
> infinito maior que outro. Outra coisa que demorei a aceitar,mesmo vendo a
> bijeção, era que os inteiros e naturais tinham a mesma cardinalidade. Na
> minha cabeça, os inteiros têm todos os naturais ainda sobram os negativos,
> como é igual?
> Hoje, depois de velho, arrumei uma enteada, que muito me pergunta e estou
> enrolado. Para dar um ar de superioridade, questionei se conhecia os
> inteiros de Gaus, que 5 não era primo nos inteiros de Gaus. Estrepei-me, a
> danada foi pesquisar e me questiona sobre o que não tenho um domínio pleno.
> Em suma, como apresentei a ela os postulados de Peano para a
> caracterização dos Naturais, ela me cobra por algo semelhante para os
> Inteiros, e não sei responder.
> HELP! SOCORRO! AU SECOURS! AYUDA! AIUTO! HILFE!
> Cordialmente,
> PJMS
>
> --
> Esta mensagem foi verificada pelo sistema de antivírus e
> acredita-se estar livre de perigo.

-- 
Esta mensagem foi verificada pelo sistema de antiv�rus e
 acredita-se estar livre de perigo.



Re: [obm-l] problema de probabilidade

2022-11-09 Por tôpico Anderson Torres
Em ter, 8 de nov de 2022 21:55, Ralph Costa Teixeira 
escreveu:

> Mis ou menos... O que faltou foi a hipótese exata da distribuição de
> probabilidade dos aniversários.
>
> Se a gente supõe que cada mês tem os mesmos 1/12 de chance para cada
> aluno, e que os meses são independentes entre si, sim,
> p=12/12^2=1/12~8.3%.
>
> Agora, talvez um modelo um pouco mais preciso seria supor que cada DIA do
> ano tem a mesma probabilidade (e que são independentes entre si). Isto
> afeta um tiquinho a resposta, porque cada mes têm um número ligeiramente
> diferente de dias! Ignorando anos bissextos (huh!?!), temos:
> -- 7 meses com 31 dias;
> -- 4 meses com 30 dias;
> -- 1 mes com 28 dias;
> Portanto, seria um pouco mais "realista" usar:
> p=(7*31^2+4*30^2+28^2)/(365^2) ~ 8.34003%
>
> Eu ponho esse "realista" bem entre aspas; primeiro, porque eu ignorei
> anos bissextos (fique à vontade para inclui-los e refazer a conta :D :D
> :D); mas a hipótese de que todos os dias do ano tem a mesma probabilidade
> não é tão realista quanto parece! Existe uma certa "concentração" de
> aniversários em determinadas épocas do ano... mas, sem dados exatos sobre
> como seja a tal concentração, o melhor que podemos fazer seria uma das
> estimativas acima.
>

Em uma turma com tão pouca gente, eu acho que considerações como "a
concentração de pessoas concebidas no Carnaval" podem ser ignoradas para um
problema tão simples. E, pelo que se nota, a conta mais limpa dá uma
diferença minúscula, 0,01%. Desconheço aplicação tão precisa na prática.


> Ainda tem um segundo problema sutil: *mesmo que todos os dias tivessem a
> mesma probabilidade, talvez n*ã*o seja 100% correto supor que os
> aniversários dos alunos da mesma turma do CMBel sejam independentes*! Por
> exemplo, existe uma probabilidade maior que zero de ter gêmeos numa mesma
> turma (comum uma família com gêmeos colocá-los na mesma escola), o que
> afeta a independência dos dados, e muda um pouquinho aqueles 8.3% (para
> cima)... sem uma estimativa desta probabilidade de ter gêmeos na mesma
> turma, não conseguimos calcular a resposta "exata".
>
> Isto tudo dito... em quase qualquer problema de probabilidade a gente vai
> ter que fazer ALGUMA hipótese simplificadora para poder sair do lugar.
> Assim, eu diria que o problema não está 100% bem posto, mas não acho
> ridículo fazer uma das hipóteses simplificadoras acima que levam a 8.3%
> ou 8.34003% (e a diferença me parece tão pequena que eu aceitaria ambas as
> respostas como corretas, desde que as hipóteses utilizadas em cada caso
> fossem citadas).
>
> Abraço, Ralph.
>
> On Tue, Nov 8, 2022 at 3:07 PM Luis Paulo  wrote:
>
>> Prezados, o problema abaixo está bem posto?
>>
>> Uma turma do CMBel tem 25 alunos. Escolhendo-se aleatoriamente dois
>> estudantes dessa turma, qual a probabilidade de eles façam aniversário no
>> mesmo mês?
>>
>> A resposta da banca: 1/12.
>>
>>
>>
>> --
>> Esta mensagem foi verificada pelo sistema de antivírus e
>> acredita-se estar livre de perigo.
>
>
> --
> Esta mensagem foi verificada pelo sistema de antivírus e
> acredita-se estar livre de perigo.

-- 
Esta mensagem foi verificada pelo sistema de antiv�rus e
 acredita-se estar livre de perigo.



Re: [obm-l] integrais

2022-10-05 Por tôpico Anderson Torres
É spam??

Em ter, 4 de out de 2022 15:48, carlos h Souza 
escreveu:

>

-- 
Esta mensagem foi verificada pelo sistema de antiv�rus e
 acredita-se estar livre de perigo.



[obm-l] Re: [obm-l] Perguntas mínimas no Tabuleiro

2022-09-14 Por tôpico Anderson Torres
Em ter, 13 de set de 2022 22:59, Jeferson Almir 
escreveu:

> Os números de 1 a 49 são arbitrariamente dispostos num tabuleiro quadrado
> 7x7 . Podemos escolher qualquer quadrado composto de múltiplas células e
> perguntar quais números estão contidos nele. Ao menos quantas perguntas são
> necessárias para determinarmos a configuração exata dos números?
>
> Alguém tem uma ideia ótima ??
>

tenho não, mas para o caso 3x3 a resposta é no máximo 4. Perguntando quem
são os elementos dos quadradinhos 2x2, é possível obter quem são os 9 caras
usando uniões e intersecções.

com isso já podemos recursivamente cortar o quadrado 7x7 em quadrados 3x3.
Se fosse 9x9, isso daria no máximo 24.

vou pensar mais em casa.


>
> --
> Esta mensagem foi verificada pelo sistema de antivírus e
> acredita-se estar livre de perigo.

-- 
Esta mensagem foi verificada pelo sistema de antiv�rus e
 acredita-se estar livre de perigo.



[obm-l] Re: [obm-l] Fwd: Módulo

2022-08-11 Por tôpico Anderson Torres
Em qui, 11 de ago de 2022 16:12, Esaú Gomes  escreveu:

> Alguém poderia me falar o que estudar mais especificamente na questão
> abaixo?
>
> Para quais valores naturais  de *n* e *x*, existe solução
> 2^n = 3x + 1.
>

  Provas antigas.

Esses problemas são resolvidos geralmente apelando para fatos padrão de
congruências, em especial potenciação, ordem etc.

E, no geral, a melhor maneira de entender e aplicar estes fatos é mediante
treino, treino e mais treino.

-- 
> Esta mensagem foi verificada pelo sistema de antivírus e
> acredita-se estar livre de perigo.

-- 
Esta mensagem foi verificada pelo sistema de antiv�rus e
 acredita-se estar livre de perigo.



[obm-l] Re: [obm-l] Função phi de Euler

2022-07-15 Por tôpico Anderson Torres
Em qui, 14 de jul de 2022 11:52, Rubens Vilhena Fonseca <
rubens.vilhen...@gmail.com> escreveu:

> Saudações a todos da lista.
> É um fato que para primos p ímpares, a função de Euler phi(p)=p-1 é sempre
> um valor par.
> Os primos 7, 13, 19, 31, 37, 67, 73, 79, 97, ... tem valores pares
> múltiplos de 3.
> Existe algum caminho a tomar para determinar quando phi(p) = 3 .(2k)?
>

quem é esse k?

Agradeço qualquer solução ou  informação ou indicação de leituras sobre o
> problema.
> Att
>
>
> --
> Esta mensagem foi verificada pelo sistema de antivírus e
> acredita-se estar livre de perigo.

-- 
Esta mensagem foi verificada pelo sistema de antiv�rus e
 acredita-se estar livre de perigo.



[obm-l] Re: [obm-l] Re: [obm-l] Função phi de Euler

2022-07-15 Por tôpico Anderson Torres
Em qui, 14 de jul de 2022 12:19, Esdras Muniz 
escreveu:

> Quis dizer φ(p)=p-1.
>
> Em qui, 14 de jul de 2022 12:02, Esdras Muniz 
> escreveu:
>
>> Oi(o)=p-1, aí isso só vale se o primo for da firma 6k+1.
>>
>
phi(4+3)=7-1


>> Em qui, 14 de jul de 2022 11:52, Rubens Vilhena Fonseca <
>> rubens.vilhen...@gmail.com> escreveu:
>>
>>> Saudações a todos da lista.
>>> É um fato que para primos p ímpares, a função de Euler phi(p)=p-1 é
>>> sempre um valor par.
>>> Os primos 7, 13, 19, 31, 37, 67, 73, 79, 97, ... tem valores pares
>>> múltiplos de 3.
>>> Existe algum caminho a tomar para determinar quando phi(p) = 3 .(2k)?
>>> Agradeço qualquer solução ou  informação ou indicação de leituras sobre
>>> o  problema.
>>> Att
>>>
>>>
>>> --
>>> Esta mensagem foi verificada pelo sistema de antivírus e
>>> acredita-se estar livre de perigo.
>>
>>
> --
> Esta mensagem foi verificada pelo sistema de antivírus e
> acredita-se estar livre de perigo.

-- 
Esta mensagem foi verificada pelo sistema de antiv�rus e
 acredita-se estar livre de perigo.



[obm-l] Re: [obm-l] Série

2022-05-08 Por tôpico Anderson Torres
Em sex., 29 de abr. de 2022 às 23:09, Israel Meireles Chrisostomo
 escreveu:
>
> Alguém aí consegue calcular o limite contida no arquivo desse link logo 
> abaixo?
> https://www.overleaf.com/project/624ee701e9cd2d14986e6f48
>

Link indisponível.

obrigado...

> --
> Israel Meireles Chrisostomo
>
> --
> Esta mensagem foi verificada pelo sistema de antivírus e
> acredita-se estar livre de perigo.

-- 
Esta mensagem foi verificada pelo sistema de antiv�rus e
 acredita-se estar livre de perigo.


=
Instru��es para entrar na lista, sair da lista e usar a lista em
http://www.mat.puc-rio.br/~obmlistas/obm-l.html
=


[obm-l] Re: [obm-l] Sistema de equações lineares

2022-04-18 Por tôpico Anderson Torres
Para de spammar

Em dom., 17 de abr. de 2022 às 01:16, Felippe Coulbert Balbi
 escreveu:
>
> Eu tenho um sistema de equações lineares com 12 variaveis: x1, x2,...,x12. 
> Essas variaveis assumem valor somente no conjunto {0, 1, 1/2, 1/3}.
>
> Eu tenho 8 equações
>
> 4 equações é um sistema linear que pode ser escrito como:
>
> Ax= b
>
> A é uma matriz de 4 linhas e 12 colunas, b é uma matriz de 4 linhas e 1 coluna
>
> As outras 4 equações são:
>
> x1+x2+x3 = 1
>
> x4+x5+x6 = 1
>
> x7+x8+x9 = 1
>
> x10+x11+x12 = 1
>
> Para quais valores de A e b, esse sistema tem solucao? Quando a solucao desse 
> sistema é unica?
>
> Grato,
> Felippe
>
> --
> Esta mensagem foi verificada pelo sistema de antivírus e
> acredita-se estar livre de perigo.

-- 
Esta mensagem foi verificada pelo sistema de antiv�rus e
 acredita-se estar livre de perigo.


=
Instru��es para entrar na lista, sair da lista e usar a lista em
http://www.mat.puc-rio.br/~obmlistas/obm-l.html
=


[obm-l] Re: [obm-l] Dúvida e ajuda.

2022-04-10 Por tôpico Anderson Torres
Em sex., 8 de abr. de 2022 às 11:17, Pedro José  escreveu:
>
> Bom dia!
> Posso concluir que um número representado por uma infinidade de algarismos 
> decimais é racional se e somente se tem um período de repetições desses 
> algarismos?
> A ida é fácil se tiver o período é racional.
> Já a volta não sei se é verdade e se for há como provar?

Acho que dá para fazer isso mais algoritmicamente.

Um número da forma

0,(A)

onde A é um período de k dígitos (por óbvio, zeros à esquerda são
permitidos, como em 0,010101010101...) é essencialmente um racional da
forma A/..9 com k noves - ou melhor escrevendo, (A/(10^k-1)).

Já números da forma 0,B(A) onde B tem m dígitos são a mesma coisa que
10^(-m)*(B+A/(10^k-1)), o que, após simplificar, dá (maçaroca
qualquer)/(10^m*(10^k-1)).

Qualquer racional por definição é da forma p/q com q natural. Bastaria
demonstrar que todo natural q tem um múltiplo da forma
(10^m*(10^k-1)), o que sai de uma aplicação de Euler-Fermat ou mesmo
de casa do pombo.

(Aliás, quem foi o BR que traduziu "princípio do escaninho" para
"princípio de casa de pombo"?)

>
> Meu objetivo primário é saber se:
> 0,123456789112233445566778899111222333444555666777888999... é racional. As 
> reticências se referem ao aumento de mais um algarismo repetido a cada 
> sequência, ou seja a primeira aparição de 1 será 1, a 2a 11 a 3a 111 e assim 
> sucessivamente, o mesmo vale para os demais algarismos.
>
> Alguém poderia me ajudar?
> Grato,
> PJMS
>
> --
> Esta mensagem foi verificada pelo sistema de antivírus e
> acredita-se estar livre de perigo.

-- 
Esta mensagem foi verificada pelo sistema de antiv�rus e
 acredita-se estar livre de perigo.


=
Instru��es para entrar na lista, sair da lista e usar a lista em
http://www.mat.puc-rio.br/~obmlistas/obm-l.html
=


Re: [obm-l] Arimetica Diofanto

2022-02-02 Por tôpico Anderson Torres
Em qua., 2 de fev. de 2022 às 00:39, Carlos Gustavo Tamm de Araujo
Moreira  escreveu:
>
> Vou enviar uma solução resumida:
> Se 3^x-5^y=2, vamos testar os menores valores de y: se y=0 então 3^x=3 e x=1. 
> Se y=1 então 3^x=7, o que não dá solução inteira.
> Se y=2 então 3^x=27 e x=3. Vamos então supor y=2+b>2, o que dá 3^x>27, logo 
> x=3+a>3, e podemos escrever a equação como
> 3^3(3^a-1)=5^2(5^b-1). Como a ordem de 3 módulo 5^2 é 20, segue que 20|a, 
> logo 3^a-1 é múltiplo de 3^20-1, e portanto é múltiplo de 61.
> Como a ordem de 5 módulo 61 é 30, segue que 30|b, logo 5^b-1 é múltiplo de 
> 5^30-1, e portanto é múltiplo de 31. Como a ordem de 3
> módulo 31 é 30, segue que 30|a, logo 3^a-1 é múltiplo de 3^30-1, e portanto é 
> múltiplo de 271. Como a ordem de 5 módulo 271 é 27,
> segue que 27|b, e como 30|b segue que 54|b, logo 5^b-1 é múltiplo de 5^54-1, 
> e portanto é múltiplo de 81, absurdo, pois, como 3^a-1 não
> é múltiplo de 3,  3^3(3^a-1) não é múltiplo de 81.

Eu tinha pensado nisso, mas não sabia qual primo usar! Inicialmente
pensei em usar a ideia de ordem para descobrir esse 5^30-1 e
fatorá-lo. Mas acertar esse pingue-pongue de ordens é complicadinho...


> Abraços,
>  Gugu
> é múltiplo de 81
>
> On Fri, Jan 28, 2022 at 5:28 PM Carlos Gustavo Tamm de Araujo Moreira 
>  wrote:
>>
>> Mas acho que lá uma solução está incompleta e as outras duas erradas...
>>
>> On Fri, Jan 28, 2022 at 5:11 PM Gabriel Torkomian  wrote:
>>>
>>> https://artofproblemsolving.com/community/q1h2640462p22841017
>>> Tem no aops
>>>
>>> Em sex., 28 de jan. de 2022 10:32, Israel Meireles Chrisostomo 
>>>  escreveu:

 3^x-5^y=2

 Em sex., 28 de jan. de 2022 09:53, Esaú Gomes  
 escreveu:
>
> E qual a equação?
>
> On Wed, Jan 26, 2022 at 3:33 PM Israel Meireles Chrisostomo 
>  wrote:
>>
>> Olá pessoal, recentimente estava estudando e me deparei com uma equação 
>> diofantina.eu tentei resolve-la mas ñ sei se está correta a solução ou 
>> incompleta, vcs poderiam por favor me ajudar a fechar o argumento?ñ 
>> quero outra solução só quero fazer da minha solução uma solução 
>> top.Tenho a impressão que falta alguma coisa.
>>
>> --
>> Esta mensagem foi verificada pelo sistema de antivírus e
>> acredita-se estar livre de perigo.
>
>
> --
> Esta mensagem foi verificada pelo sistema de antivírus e
> acredita-se estar livre de perigo.


 --
 Esta mensagem foi verificada pelo sistema de antivírus e
 acredita-se estar livre de perigo.
>>>
>>>
>>> --
>>> Esta mensagem foi verificada pelo sistema de antivírus e
>>> acredita-se estar livre de perigo.
>
>
> --
> Esta mensagem foi verificada pelo sistema de antivírus e
> acredita-se estar livre de perigo.

-- 
Esta mensagem foi verificada pelo sistema de antiv�rus e
 acredita-se estar livre de perigo.


=
Instru��es para entrar na lista, sair da lista e usar a lista em
http://www.mat.puc-rio.br/~obmlistas/obm-l.html
=


Re: [obm-l] Arimetica Diofanto

2022-01-28 Por tôpico Anderson Torres
Em qua., 26 de jan. de 2022 às 15:33, Israel Meireles Chrisostomo
 escreveu:
>
> Olá pessoal, recentimente estava estudando e me deparei com uma equação 
> diofantina.eu tentei resolve-la mas ñ sei se está correta a solução ou 
> incompleta, vcs poderiam por favor me ajudar a fechar o argumento?ñ quero 
> outra solução só quero fazer da minha solução uma solução top.Tenho a 
> impressão que falta alguma coisa.
>

Que equação?

> --
> Esta mensagem foi verificada pelo sistema de antivírus e
> acredita-se estar livre de perigo.

-- 
Esta mensagem foi verificada pelo sistema de antiv�rus e
 acredita-se estar livre de perigo.


=
Instru��es para entrar na lista, sair da lista e usar a lista em
http://www.mat.puc-rio.br/~obmlistas/obm-l.html
=


Re: [obm-l] Irracional

2022-01-09 Por tôpico Anderson Torres
Em sex., 7 de jan. de 2022 às 09:21, Israel Meireles Chrisostomo
 escreveu:
>
> Olá acho que consigo provar o seguinte teorema: entre dois racionais existe 
> uma sequência de números irracionais, decrescente e crescente, com quantos 
> termos se desejar.Alguém aí se interessa por esse problema?

se é crescente E descrescente ao mesmo tempo, então é uma sequência constante.

>
> --
> Esta mensagem foi verificada pelo sistema de antivírus e
> acredita-se estar livre de perigo.

-- 
Esta mensagem foi verificada pelo sistema de antiv�rus e
 acredita-se estar livre de perigo.


=
Instru��es para entrar na lista, sair da lista e usar a lista em
http://www.mat.puc-rio.br/~obmlistas/obm-l.html
=


[obm-l] Re: [obm-l] Número de matrizes 0-1

2022-01-05 Por tôpico Anderson Torres
Em seg., 20 de dez. de 2021 às 18:58, Claudio Buffara
 escreveu:
>
> Num outro grupo, propuseram o problema de achar o número de matrizes 4x4 com 
> entradas em {0,1} e cujo determinante seja ímpar.
> Olhando mod 2, isso é equivalente a achar o número de matrizes 4x4 
> invertíveis com entradas em Z2 (o corpo com 2 elementos).
> Este é um resultado conhecido: o número de tais matrizes é 
> (2^4-1)(2^4-2)(2^4-2^2)(2^4-2^3) = 15*14*12*8 = 20.160.
> Provar isso pode ser um bom problema pra quem não conhece o "truque" (que 
> nada mais é do que usar uma caracterização alternativa de "matriz 
> invertível").
>
> Daí surgiram duas dúvidas:
> 1) Quais os valores possíveis do determinante desta matriz?
> 2) Quantas matrizes existem com cada valor possível do determinante?

Eu desconfio fortemente que não existe forma mais fácil do que fazer na raça.

>
> Não é difícil fazer um programa de computador pra calcular isso (afinal, 
> existem apenas 2^16 = 65.536 matrizes 4x4 com entradas em {0,1}).
> Mas será que há uma forma "esperta" de calcular isso?
> E que seja generalizável pra matrizes nxn?
>
> []s,
> Claudio.
>
> --
> Esta mensagem foi verificada pelo sistema de antivírus e
> acredita-se estar livre de perigo.

-- 
Esta mensagem foi verificada pelo sistema de antiv�rus e
 acredita-se estar livre de perigo.


=
Instru��es para entrar na lista, sair da lista e usar a lista em
http://www.mat.puc-rio.br/~obmlistas/obm-l.html
=


[obm-l] Re: [obm-l] como saber se uma moeda é viciada

2022-01-05 Por tôpico Anderson Torres
Em ter., 21 de dez. de 2021 às 09:16, jamil dasilva
 escreveu:
>
> Se em cem lançamentos de uma moeda a probabilidade de sair qualquer um dos 
> 2^100 resultados é a mesma,
> seria correto dizer que a moeda seria viciada se o resultado fosse CARA em 
> todas as cem vezes ?

Isso me parece bastante estranho.

Isso seria equivalente a dizer "a moeda é viciada se o resultado é
idêntico aos cem primeiros dígitos após a vírgula da expansão binária
de pi^2/6".

Eu suspeito que, se existe uma teoria sobre vício em dispositivos de
geração de número aleatório, o seu método seria bem pouco recomendado.
Algo mais certeiro seria calcular uma medida de concentração e/ou dispersão.

> --- Como explicar esse PARADOXO ? Como decidir com base em experimentos de 
> observação frequencial se uma moeda é, ou não, VICIADA ? Você acreditaria que 
> uma moeda é HONESTA se desse CARA em cem lançamentos consecutivos ?
>
> --
> Esta mensagem foi verificada pelo sistema de antivírus e
> acredita-se estar livre de perigo.

-- 
Esta mensagem foi verificada pelo sistema de antiv�rus e
 acredita-se estar livre de perigo.


=
Instru��es para entrar na lista, sair da lista e usar a lista em
http://www.mat.puc-rio.br/~obmlistas/obm-l.html
=


Re: [obm-l] CARA ou COROA com dez moedas

2022-01-04 Por tôpico Anderson Torres
Em qua., 22 de dez. de 2021 às 12:00, jamil dasilva
 escreveu:
>
> Duas pessoas disputam um CARA e COROA, jogando uma moeda honesta CEM VEZES.Um 
> deles aposta  que em todos os lançamentos ocorrerá  CARA e o outro, por sua 
> vez, aposta que ocorrerá CARA apenas  nos primeiros cinquenta lançamentos e, 
> consequentemente, cinquenta coroas nos cinquenta últimos.Qual deles tem maior 
> probabilidade de ganhar nesse CARA e COROA ?
>

Nem um nem outro.

Ambos estão apostando uma mesma coisa: se a sequência que vai sair é



ou

1100

as quais são equiprováveis.

>
>
>
> --
> Esta mensagem foi verificada pelo sistema de antivírus e
> acredita-se estar livre de perigo.

-- 
Esta mensagem foi verificada pelo sistema de antiv�rus e
 acredita-se estar livre de perigo.


=
Instru��es para entrar na lista, sair da lista e usar a lista em
http://www.mat.puc-rio.br/~obmlistas/obm-l.html
=


[obm-l] Re: [obm-l] Números de tentativas

2021-12-13 Por tôpico Anderson Torres
Em seg., 13 de dez. de 2021 às 10:00, Jeferson Almir
 escreveu:
>
> Amigos peço ajuda nessa questão.
>
> Tem uma senha de 3 digitos
> (Qualquer digito  de 0 a 9)
> E nos temos um dispositivo
> Que compara a senha
> Com um número que escolhemos
> E retorna não se tem todos os digitos diferentes da senha
> E retorna quase se tem pelo menos 1 digito coincidente com a senha
> Qual é o menor numero de tentativas que precisamos usar esse dispositivo tal 
> que podemos descobrir a senha com certeza, independente de qual ela seja?

Por ora eu vou fazer uma tentativa.

Se fosse uma senha de um dígito, temos 10 tentativas.

Se fossem dois, bem, vamos pensar um pouco. Inicialmente não sei o que
fazer, vou simplesmente chutar AB

1. A máquina diz "acertou 2". 1 tentativa
2. A máquina diz "acertou 1". Aqui reduzimos o conjunto de tentativas
em 18 (A?- 9 tentativas; B? -  9 tentativas)
3. A máquina diz "acertou 0". Aqui piora: 9*9=81 tentativas sobrando.

No caso mais desfavorável, em 5 tentativas dá para limpar o conjunto.

Claro, isso não prova nada ainda.

>
> --
> Esta mensagem foi verificada pelo sistema de antivírus e
> acredita-se estar livre de perigo.

-- 
Esta mensagem foi verificada pelo sistema de antiv�rus e
 acredita-se estar livre de perigo.


=
Instru��es para entrar na lista, sair da lista e usar a lista em
http://www.mat.puc-rio.br/~obmlistas/obm-l.html
=


[obm-l] Re: [obm-l] Valor máximo

2021-11-29 Por tôpico Anderson Torres
Em ter., 23 de nov. de 2021 às 21:54, marcone augusto araújo borges
 escreveu:
>
> Se a, b e c são positivos e a^2+b^2+c^2 = 1, qual o valor máximo de 
> (1-a)(1-b)(1-c)?

Acho, só acho, que dá para simplesmente fazer assim:

Se fixarmos c, temos que determinar o máximo de (1-a)(1-b) dado que
a^2+b^2=Z^2(=1-c^2) Minha suspeita levemente mal fundada é que isso é
máximo quando a e b são iguais. Com isso bastaria maximizar uma certa
função em Z.

Outra forma seria escrever a = cosF, b = sinF sinG, c=sinF cosG e usar
um pouco de análise de uma variável, por exemplo fixando G e
verificando F.

> Desde já agradeço
>
> --
> Esta mensagem foi verificada pelo sistema de antivírus e
> acredita-se estar livre de perigo.

-- 
Esta mensagem foi verificada pelo sistema de antiv�rus e
 acredita-se estar livre de perigo.


=
Instru��es para entrar na lista, sair da lista e usar a lista em
http://www.mat.puc-rio.br/~obmlistas/obm-l.html
=


Re: [obm-l] Lei dos cossenos e Lei dos senos

2021-10-15 Por tôpico Anderson Torres
Smells like argumentação circular.

Em qui., 30 de set. de 2021 às 17:09, Israel Meireles Chrisostomo
 escreveu:
>
> Olá, ultimamente fiz uma prova para lei dos cossenos e senos, mas não sei se 
> está correta, alguém poderia por favor  me ajudar na correção?
> O link com a solução segue abaixo
> https://www.overleaf.com/read/zfcqwwmgxnrt
>
> --
> Esta mensagem foi verificada pelo sistema de antivírus e
> acredita-se estar livre de perigo.

-- 
Esta mensagem foi verificada pelo sistema de antiv�rus e
 acredita-se estar livre de perigo.


=
Instru��es para entrar na lista, sair da lista e usar a lista em
http://www.mat.puc-rio.br/~obmlistas/obm-l.html
=


[obm-l] Re: [obm-l] Re: [obm-l] Re: [obm-l] Re: matemática discreta

2021-09-20 Por tôpico Anderson Torres
Não consegui entender esse texto.

Em seg., 20 de set. de 2021 às 22:37, Israel Meireles Chrisostomo
 escreveu:
>
> Obrigado
>
> Em seg, 20 de set de 2021 22:00, Israel Meireles Chrisostomo 
>  escreveu:
>>
>> Tome n maior que n
>>
>> Em seg, 20 de set de 2021 20:49, Marcelo Salhab Brogliato 
>>  escreveu:
>>>
>>> Oi Israel,
>>>
>>> Não consegui entender a questão.
>>>
>>> Exemplo:
>>>
>>> n = 10, m = 3, Fib(10 - 3 + 1) = Fib(8) = 21
>>>
>>> (alpha**(2*n)) / (alpha**(n - m)) = alpha**(n + m) = 521.0019193787257
>>>
>>> Pela sua igualdade, alpha**(n + m) deveria ser 1/21, correto?
>>>
>>> Abraços,
>>> Marcelo
>>>
>>> Il giorno lun 20 set 2021 alle ore 15:54 Israel Meireles Chrisostomo 
>>>  ha scritto:

 já tentei de tudo, por favor me ajudem.

 Em seg., 20 de set. de 2021 às 19:39, Israel Meireles Chrisostomo 
  escreveu:
>
> Alguém poderia resolver o problema no link abaixo?
>
> https://mathoverflow.net/questions/404417/alpha2n-fracf-n-m1-alphan-m-1-how-to-prove-that-equality-is-true
>
> --
> Israel Meireles Chrisostomo



 --
 Israel Meireles Chrisostomo

 --
 Esta mensagem foi verificada pelo sistema de antivírus e
 acredita-se estar livre de perigo.
>>>
>>>
>>> --
>>> Esta mensagem foi verificada pelo sistema de antivírus e
>>> acredita-se estar livre de perigo.
>
>
> --
> Esta mensagem foi verificada pelo sistema de antivírus e
> acredita-se estar livre de perigo.

-- 
Esta mensagem foi verificada pelo sistema de antiv�rus e
 acredita-se estar livre de perigo.


=
Instru��es para entrar na lista, sair da lista e usar a lista em
http://www.mat.puc-rio.br/~obmlistas/obm-l.html
=


[obm-l] Re: [obm-l] Ajuda numa questão da OBM 1987

2021-08-21 Por tôpico Anderson Torres
Em ter., 20 de jul. de 2021 às 18:25, Prof. Douglas Oliveira
 escreveu:
>
> Tem-se um bolo em forma de prisma triangular, cuja base está em um plano 
> horizontal. Dois indivíduos vão dividir o bolo de acordo com a seguinte 
> regra: o primeiro escolhe um ponto na base superior do bolo e o segundo corta 
> o bolo por um plano vertical à sua escolha, passando porém pelo ponto 
> escolhido, e seleciona para si um dos pedaços em que dividiu o bolo. Qual 
> deve ser a estratégia para o primeiro e qual deve ser a fração do volume do 
> bolo que ele espera obter?

Primeira dica: tente resolver o mesmo problema, mas para um triângulo
equilátero de lado 100.
Afinal de contas, uma transformação afim leva isso para qualquer triângulo.

>
> Abraço do Douglas.
>
> --
> Esta mensagem foi verificada pelo sistema de antivírus e
> acredita-se estar livre de perigo.

-- 
Esta mensagem foi verificada pelo sistema de antiv�rus e
 acredita-se estar livre de perigo.


=
Instru��es para entrar na lista, sair da lista e usar a lista em
http://www.mat.puc-rio.br/~obmlistas/obm-l.html
=


[obm-l] Re: [obm-l] "números biquadrados"

2021-08-13 Por tôpico Anderson Torres
Em qui, 12 de ago de 2021 21:17, marcone augusto araújo borges <
marconeborge...@hotmail.com> escreveu:

> 1233 = 12^2 + 33^2
> Em uma prova da bom nível 2, o número 1233 foi apresentado como
> "biquadrado" e foi pedido outro número biquadrado
> Eu pensei
> A^2+ B^2 = 100A + B
> A^2 - 100A + B^2 - B = 0
> Seriam dois valores para A cuja soma é 100, então se um deles é 12 o outro
> é 88
> Observei que 8833 = 88^2 + 33^2
> Se não fosse dado o 1233, daria para calcular os dois números...
> Como resolver A^2 + B^2 = 100A + B, com A e B inteiros positivos?
>

A^2 - 100A + B^2 - B = 0

4A^2 - 2*100*2A + 4B^2 - 2*2B = 0

(2A)^2 - 2*100*2A + 100^2 + (2B)^2 - 2*2B +1 = 100^2+1

(2A-100)^2+(2B-1)^2 = 10001

Agora é calcular mecanicamente todas as possibilidades para A e B.



> --
> Esta mensagem foi verificada pelo sistema de antivírus e
>


> acredita-se estar livre de perigo.
>

-- 
Esta mensagem foi verificada pelo sistema de antiv�rus e
 acredita-se estar livre de perigo.



[obm-l] Re: [obm-l] Re: [obm-l] Álgebra

2021-07-27 Por tôpico Anderson Torres
Em dom., 25 de jul. de 2021 às 15:23, Ralph Costa Teixeira
 escreveu:
>
> Sem ser muito formal: (a,b) e (c,d) sao dois vetores do plano, unitários e 
> ortogonais. Ou seja, um deles eh igual ao outro girado de 90 graus. Assim 
> (c,d)=(-b,a) ou (c,d)=(b,-a). De um jeito ou de outro, cd=-ab, ou seja, 
> resposta 0.
>
> On Sun, Jul 25, 2021 at 10:03 AM marcone augusto araújo borges 
>  wrote:
>>
>> a, b, c, d são números reais tais que a^2+b^2 = c^2 + d^2 = 1, ac + bd = 0. 
>> Calcule ab + cd
>> Desde já agradeço

Poderíamos escrever a=sinX, c=cosY. Assim sendo, b=cosX, d=sinY e daí
0=ac+bd=sinXcosY + cosXsinY = sin(X+Y), assim podemos usar X=-Y e daí
sinXcosX+sinYcosY = sinXcosX-sinXcosX=0
>>
>> --
>> Esta mensagem foi verificada pelo sistema de antivírus e
>> acredita-se estar livre de perigo.
>
>
> --
> Esta mensagem foi verificada pelo sistema de antivírus e
> acredita-se estar livre de perigo.

-- 
Esta mensagem foi verificada pelo sistema de antiv�rus e
 acredita-se estar livre de perigo.


=
Instru��es para entrar na lista, sair da lista e usar a lista em
http://www.mat.puc-rio.br/~obmlistas/obm-l.html
=


Re: [obm-l] Limites

2021-06-30 Por tôpico Anderson Torres
Em sex., 25 de jun. de 2021 às 23:38, Israel Meireles Chrisostomo
 escreveu:
>
> Alguém aí consegue provar o teorema do confronto?Em caso afirmativo por favor 
>  prove-o
>

??

> --
> Esta mensagem foi verificada pelo sistema de antivírus e
> acredita-se estar livre de perigo.

-- 
Esta mensagem foi verificada pelo sistema de antiv�rus e
 acredita-se estar livre de perigo.


=
Instru��es para entrar na lista, sair da lista e usar a lista em
http://www.mat.puc-rio.br/~obmlistas/obm-l.html
=


[obm-l] Re: [obm-l] base de numeração

2021-05-19 Por tôpico Anderson Torres
Em ter., 27 de abr. de 2021 às 17:40, Daniel Quevedo
 escreveu:
>
> Os oito últimos algarismos do número 27^1986 quando escrito na base 2 são:
> a) 11011001
> b) 11011101
> c) 1001
> d) 11011011
> e) 10011001
>
> gab: A

Calcule o resto da divisão de 27^1986 por 2^8, depois converta para binário.

>
> --
> Fiscal: Daniel Quevedo
>
> --
> Esta mensagem foi verificada pelo sistema de antivírus e
> acredita-se estar livre de perigo.

-- 
Esta mensagem foi verificada pelo sistema de antiv�rus e
 acredita-se estar livre de perigo.


=
Instru��es para entrar na lista, sair da lista e usar a lista em
http://www.mat.puc-rio.br/~obmlistas/obm-l.html
=


[obm-l] Re: [obm-l] Re: [obm-l] Re: [obm-l] Função

2021-05-19 Por tôpico Anderson Torres
Em seg., 26 de abr. de 2021 às 17:18, Israel Meireles Chrisostomo
 escreveu:
>
> Mas aí então a+bi e b+ai são os mesmos números

Não são.

4+5i e 5+4i são diferentes, e 4+5i < 5+4i por essas regras.

>
> Em seg, 26 de abr de 2021 13:36, Anderson Torres 
>  escreveu:
>>
>> Em qui., 22 de abr. de 2021 às 07:19, Israel Meireles Chrisostomo
>>  escreveu:
>> >
>> > Me desculpem se eu estou falando bobagem, mas considere uma função com 
>> > domínio complexo, então essa função não pode ser bijetora, pois toda 
>> > função bijetora ou é crescente ou é decrescente, mas não há ordem nos 
>> > complexos

Você não entendeu nada aqui, suponho. Primeiramente, funções não são
coisas limitadas a números.

Segundamente, quando usamos esse teorema de que funções contínuas são
monótonas, é óbvio que estamos supondo de antemão que estamos
trabalhando com um sistema numérico que admita a ideia de ordem.
Especialmente, a de um corpo ordenado completo.

Por exemplo, não faz sentido falar de "continuidade" quando se fala de
funções de naturais para naturais, porque números naturais não formam
um sistema numérico contínuo.

>>
>> Não é correto dizer que não existe ordem nos complexos. É só atribuir
>> o seguinte: o complexo A é maior que o complexo B se e somente se ou o
>> módulo de A é maior que o de B ou os módulos são iguais mas o
>> argumento de A é maior que o de B (tomando este módulo no intervalo de
>> 0 a tau).
>>
>> >
>> > --
>> > Esta mensagem foi verificada pelo sistema de antivírus e
>> > acredita-se estar livre de perigo.
>>
>> --
>> Esta mensagem foi verificada pelo sistema de antivírus e
>>  acredita-se estar livre de perigo.
>>
>>
>> =
>> Instru�ões para entrar na lista, sair da lista e usar a lista em
>> http://www.mat.puc-rio.br/~obmlistas/obm-l.html
>> =
>
>
> --
> Esta mensagem foi verificada pelo sistema de antivírus e
> acredita-se estar livre de perigo.

-- 
Esta mensagem foi verificada pelo sistema de antiv�rus e
 acredita-se estar livre de perigo.


=
Instru��es para entrar na lista, sair da lista e usar a lista em
http://www.mat.puc-rio.br/~obmlistas/obm-l.html
=


Re: [obm-l] Produto infinito do seno

2021-05-19 Por tôpico Anderson Torres
Em seg., 17 de mai. de 2021 às 18:58, Israel Meireles Chrisostomo
 escreveu:
>
> Alguém aí sabe quantas provas existem para se verificar q d fato o produto 
> infinito do seno é verdadeiro?Eu tenho uma.
>

HEIN

> --
> Esta mensagem foi verificada pelo sistema de antivírus e
> acredita-se estar livre de perigo.

-- 
Esta mensagem foi verificada pelo sistema de antiv�rus e
 acredita-se estar livre de perigo.


=
Instru��es para entrar na lista, sair da lista e usar a lista em
http://www.mat.puc-rio.br/~obmlistas/obm-l.html
=


[obm-l] Re: [obm-l] Função

2021-04-26 Por tôpico Anderson Torres
Em qui., 22 de abr. de 2021 às 07:19, Israel Meireles Chrisostomo
 escreveu:
>
> Me desculpem se eu estou falando bobagem, mas considere uma função com 
> domínio complexo, então essa função não pode ser bijetora, pois toda função 
> bijetora ou é crescente ou é decrescente, mas não há ordem nos complexos

Não é correto dizer que não existe ordem nos complexos. É só atribuir
o seguinte: o complexo A é maior que o complexo B se e somente se ou o
módulo de A é maior que o de B ou os módulos são iguais mas o
argumento de A é maior que o de B (tomando este módulo no intervalo de
0 a tau).

>
> --
> Esta mensagem foi verificada pelo sistema de antivírus e
> acredita-se estar livre de perigo.

-- 
Esta mensagem foi verificada pelo sistema de antiv�rus e
 acredita-se estar livre de perigo.


=
Instru��es para entrar na lista, sair da lista e usar a lista em
http://www.mat.puc-rio.br/~obmlistas/obm-l.html
=


Re: [obm-l]

2021-04-26 Por tôpico Anderson Torres
Em dom., 25 de abr. de 2021 às 14:34, Artur Costa Steiner
 escreveu:
>
> Oh, no meu email anterior, onde se
> lê raiz(3), leia-se raiz_cúbica(2). Tô fazendo um tratamento na vista e ando 
> com dificuldade para digitar num celular.
> Um cara de 69 anos como eu não deveria mais participar deste grupo

Como se estivéssemos disputando uma Medalha Fields!

>
> Artur
>
>
>
> Em dom., 25 de abr. de 2021 14:16, Artur Costa Steiner 
>  escreveu:
>>
>> raiz(2)) e raiz(3) são inteiros algébricos, visto serem raízes de x^2 - 2 e 
>> x^3 - 2, respectivamente. Segundo um clássico teorema da Teoria dos Números, 
>> a soma de dois inteiros algébricos é inteira algébrica. E um inteiro 
>> algébrico é racional se, e somente se, for inteiro. Como, conforme já 
>> comentado, este não é o caso de raiz(2) + raiz(3), segue-se que está soma é 
>> irracional.
>>
>> Abraços
>> Artur
>>
>>
>> Em sex., 23 de abr. de 2021 17:43, Marcos Martinelli 
>>  escreveu:
>>>
>>> Legal, Matheus.
>>>
>>> Minha ideia foi encontrar um polinômio em m.n (m = raiz(2) e 
>>> n=raiz_cúbica(2)) de coeficientes racionais. Pra isso desenvolvi m^k + n^k 
>>> (k >= 0) até k=6 e encontrei um de grau 6 com coeficientes dependendo só de 
>>> m+n.
>>>
>>> Se m+n for racional, usei o fato de se a + beta (a racional e beta 
>>> irracional com beta^j  também irracional (1=< j <= grau do polinômio- 1) 
>>> for raiz desse polinômio então a - beta também seria.
>>>
>>> Mas essa sua ficou bem elegante.
>>>
>>> Brigado.
>>>
>>> Em sex., 23 de abr. de 2021 às 17:18, Matheus Secco 
>>>  escreveu:

 Oi, Marcos. Não é difícil verificar que raiz(2) + raiz_cubica(2) é uma 
 raiz do polinômio x^6 - 6 x^4 - 4 x^3 + 12 x^2 - 24 x - 4. Com isso, pelo 
 teorema das raízes racionais, se raiz(2) + raiz_cubica(2) fosse racional, 
 teria que ser um inteiro e é fácil verificar que 2 < raiz(2) + 
 raiz_cubica(2) < 3.

 Abraços

 On Fri, Apr 23, 2021 at 4:43 PM Marcos Martinelli 
  wrote:
>
> Opa, pessoal. Pensei nos últimos dias no problema seguinte. Cheguei a uma 
> solução um pouco mais genérica, mas me deu trabalho. Gostaria de estudar 
> outras abordagens.
>
> Problema) Prove que raiz (2) + raiz_cúbica (2) é irracional.
>
> Na sequência posto um rascunho do que pensei.
>
> Obrigado.
>
> --
> Esta mensagem foi verificada pelo sistema de antivírus e
> acredita-se estar livre de perigo.


 --
 Esta mensagem foi verificada pelo sistema de antivírus e
 acredita-se estar livre de perigo.
>>>
>>>
>>> --
>>> Esta mensagem foi verificada pelo sistema de antivírus e
>>> acredita-se estar livre de perigo.
>
>
> --
> Esta mensagem foi verificada pelo sistema de antivírus e
> acredita-se estar livre de perigo.

-- 
Esta mensagem foi verificada pelo sistema de antiv�rus e
 acredita-se estar livre de perigo.


=
Instru��es para entrar na lista, sair da lista e usar a lista em
http://www.mat.puc-rio.br/~obmlistas/obm-l.html
=


Re: [obm-l] Magnitude

2021-04-26 Por tôpico Anderson Torres
Em seg., 26 de abr. de 2021 às 00:16, Pedro Lazéra
 escreveu:
>
> Boa noite, Maikel.
>
> A quantidade de algarismos de um número x (na base 10) é "1 + piso de 
> log(x)", em que "log" é a função logaritmo na base 10. Você pode verificar 
> isso assim: 10^n, para n inteiro >= 0, é o menor número do mundo com n+1 
> algarismos. Além disso, log(10^n) = n. Por fim, log é uma função crescente.
>
> Usando que log(a*b) = log(a) + log(b), uma solução computeira é fazer um 
> programa que calcula log(i), para i inteiro entre 1 e 100, e soma esses 
> valores a uma variável s inicializada com o valor ZERO.
>
> Uma solução computeira (provavelmente) errada é calcular x = 100! e depois 
> achar o log(x). Esse valor x não cabe nas estruturas de dados que a maioria 
> das linguagens usa para representar números.
>
> Bom, a soma dá 157.97, conforme o Anderson Torres falou antes de mim. A gente 
> sabe que os computadores até erram (truncam) o valor exato de log(x), mas o 
> erro é bem pequeno e só estamos somando 100 aplicações da função log, daí 
> sabemos que esse 157.97 pode até estar errado, mas é por muito pouco (menos 
> do que 0,01, por exemplo).

Na pior hipótese, é só pegar cada log com 3 casas de precisão. Isso
significa que estaríamos somando os erros da terceira casa depois da
vírgula, que vezes 100 só chegariam na primeira casa depois da vírgula
no máximo: se cada erro fosse 0.009, vezes 100 isso daria um erro de
0.9.
Então nem tem com o que se preocupar aqui.

>
> Finalmente, 100! tem 1 + piso(157.97) = 158 algarismos.
>
> Abraços,
> Pedro
>
> On Sun, Apr 11, 2021 at 12:29 AM Anderson Torres 
>  wrote:
>>
>> Em sáb., 3 de abr. de 2021 às 01:13, Maikel Andril Marcelino
>>  escreveu:
>> >
>> > Quantos algarismos tem o número (100!) ?
>>
>> Em outras palavras, qual é o log(100!)/log(10). O Google me diz que
>> isso é 157,97 - logo, 158 dígitos.
>>
>> >
>> >
>> > Atenciosamente,
>> >
>> > Maikel Andril Marcelino
>> > Assistente de Aluno - Biblioteca - Ramal: 7616
>> > Coordenadoria de Apoio Acadêmico - COAPAC/IFRN-SPP
>> > Instituto Federal do Rio Grande do Norte
>> > Campus São Paulo do Potengi
>> >
>> > +55 (84) 8851-3451
>> >
>> > --
>> > Esta mensagem foi verificada pelo sistema de antivírus e
>> > acredita-se estar livre de perigo.
>>
>> --
>> Esta mensagem foi verificada pelo sistema de antivírus e
>>  acredita-se estar livre de perigo.
>>
>>
>> =
>> Instru�ões para entrar na lista, sair da lista e usar a lista em
>> http://www.mat.puc-rio.br/~obmlistas/obm-l.html
>> =
>
>
> --
> Esta mensagem foi verificada pelo sistema de antivírus e
> acredita-se estar livre de perigo.

-- 
Esta mensagem foi verificada pelo sistema de antiv�rus e
 acredita-se estar livre de perigo.


=
Instru��es para entrar na lista, sair da lista e usar a lista em
http://www.mat.puc-rio.br/~obmlistas/obm-l.html
=


[obm-l] Re: [obm-l] Questão sobre desigualdades

2021-04-14 Por tôpico Anderson Torres
Em qua., 14 de abr. de 2021 às 15:54, Carlos Monteiro
 escreveu:
>
> Encontre os valores máximo e mínimo da expressão:  x/(x^2+1) + y/(y^2+1) + 
> z/(z^2+1) , onde x, y e z são números reais que satisfazem x+y+z = 1.
>
>

Verifica-se que 3(12x+1)/50 >= x/(x^2+1), e assim o valor máximo é 3/10

>
>
> --
> Esta mensagem foi verificada pelo sistema de antivírus e
> acredita-se estar livre de perigo.

-- 
Esta mensagem foi verificada pelo sistema de antiv�rus e
 acredita-se estar livre de perigo.


=
Instru��es para entrar na lista, sair da lista e usar a lista em
http://www.mat.puc-rio.br/~obmlistas/obm-l.html
=


Re: [obm-l] transcendencia

2021-04-10 Por tôpico Anderson Torres
Em qui., 1 de abr. de 2021 às 18:02, Israel Meireles Chrisostomo
 escreveu:
>
>
> Como posso provar que se u é um número transcendente e a_k são números 
> algébricos quaisquer, para todo k natural, então  ua_0+ ua_1+ ua_2+...+ ua_n 
> não pode ser igual a zero.

Fatorando U.

> --
> Israel Meireles Chrisostomo
>
> --
> Esta mensagem foi verificada pelo sistema de antivírus e
> acredita-se estar livre de perigo.

-- 
Esta mensagem foi verificada pelo sistema de antiv�rus e
 acredita-se estar livre de perigo.


=
Instru��es para entrar na lista, sair da lista e usar a lista em
http://www.mat.puc-rio.br/~obmlistas/obm-l.html
=


Re: [obm-l] Magnitude

2021-04-10 Por tôpico Anderson Torres
Em sáb., 3 de abr. de 2021 às 01:13, Maikel Andril Marcelino
 escreveu:
>
> Quantos algarismos tem o número (100!) ?

Em outras palavras, qual é o log(100!)/log(10). O Google me diz que
isso é 157,97 - logo, 158 dígitos.

>
>
> Atenciosamente,
>
> Maikel Andril Marcelino
> Assistente de Aluno - Biblioteca - Ramal: 7616
> Coordenadoria de Apoio Acadêmico - COAPAC/IFRN-SPP
> Instituto Federal do Rio Grande do Norte
> Campus São Paulo do Potengi
>
> +55 (84) 8851-3451
>
> --
> Esta mensagem foi verificada pelo sistema de antivírus e
> acredita-se estar livre de perigo.

-- 
Esta mensagem foi verificada pelo sistema de antiv�rus e
 acredita-se estar livre de perigo.


=
Instru��es para entrar na lista, sair da lista e usar a lista em
http://www.mat.puc-rio.br/~obmlistas/obm-l.html
=


[obm-l] Re: [obm-l] Algébricos

2021-04-10 Por tôpico Anderson Torres
Em seg., 5 de abr. de 2021 às 21:57, Israel Meireles Chrisostomo
 escreveu:
>
> O número i é algebricamente dependente de pi?
>

O que é algebricamente dependente?

> --
> Esta mensagem foi verificada pelo sistema de antivírus e
> acredita-se estar livre de perigo.

-- 
Esta mensagem foi verificada pelo sistema de antiv�rus e
 acredita-se estar livre de perigo.


=
Instru��es para entrar na lista, sair da lista e usar a lista em
http://www.mat.puc-rio.br/~obmlistas/obm-l.html
=


[obm-l] Re: [obm-l] Transcendência

2021-03-30 Por tôpico Anderson Torres
Em seg., 29 de mar. de 2021 às 21:07, Israel Meireles Chrisostomo
 escreveu:
>
> Estou desconfiado de um resultado, mas não sei como prová-lo.o resultado é o 
> seguinte: dados dois números a,b  transcendentes e algebricamente dependentes 
> e c um número, se a,b e c são algebricamente dependentes, então c é 
> transcendente.é verdade esse resultado?se sim, como posso prová-lo?
>

Como exatamente dois números são algebricamente dependentes?

Você parece estar perdendo tempo em algo.

> --
> Esta mensagem foi verificada pelo sistema de antivírus e
> acredita-se estar livre de perigo.

-- 
Esta mensagem foi verificada pelo sistema de antiv�rus e
 acredita-se estar livre de perigo.


=
Instru��es para entrar na lista, sair da lista e usar a lista em
http://www.mat.puc-rio.br/~obmlistas/obm-l.html
=


Re: [obm-l] Transcende

2021-03-30 Por tôpico Anderson Torres
Em seg., 29 de mar. de 2021 às 23:17, Israel Meireles Chrisostomo
 escreveu:
>
> Como provar que dados u
> algébrico e v transcendente, qualquer combinação linear racional de u e v, 
> também será transcendente.

Sério?

Combinações lineares de algébricos são algébricas.
Se você não sabe disso ainda, então tem que voltar a estudar.

E combinações lineares são transitivas, no sentido que se C é
combinação de A e B então A é combinação de B e C.

>
> --
> Esta mensagem foi verificada pelo sistema de antivírus e
> acredita-se estar livre de perigo.

-- 
Esta mensagem foi verificada pelo sistema de antiv�rus e
 acredita-se estar livre de perigo.


=
Instru��es para entrar na lista, sair da lista e usar a lista em
http://www.mat.puc-rio.br/~obmlistas/obm-l.html
=


Re: [obm-l] Amigo secreto ENEM

2021-02-23 Por tôpico Anderson Torres
Em qui., 28 de jan. de 2021 às 13:15, Luiz Antonio Rodrigues
 escreveu:
>
> Olá, pessoal!
> Boa tarde!
> Estou acompanhando com interesse a discussão, mas gostaria de pedir uma 
> indicação de site ou outro material que trate de permutações caóticas.

Procure por derangements no Google.

> Muito obrigado!
> Abraços!
> Luiz
>
> Em qui, 28 de jan de 2021 11:38 AM, Arthur Queiroz  
> escreveu:
>>
>> Uma pergunta: você assume que o número de sorteios é !10. Mas e se, em meio 
>> ao sorteio, nossa permutação caótica seja tal que seja formado um ciclo 
>> indesejado? Digamos A->B->C->A. Como o sorteio continuará nesse caso? Será 
>> escolhida aleatoriamente uma pessoa de fora do ciclo para continuar? Isso 
>> não afetaria esse !10?
>>
>> Em ter, 26 de jan de 2021 17:26, Ralph Costa Teixeira  
>> escreveu:
>>>
>>> Deixa eu copiar o que escrevi em outro lugar... :D :D
>>>
>>> Primeiro: não fica claro do enunciado se "auto-sorteios" (alguém sortear o 
>>> próprio nome) são permitidos ou não, e isto ALTERA a resposta. :(
>>>
>>> Vejamos possíveis respostas corretas:
>>>
>>> ---///---
>>>
>>> SE AUTO-SORTEIOS FOREM PERMITIDOS:
>>> Em resumo, temos 1/10 de chance de A iniciar o sorteio, e 1/10 de chance de 
>>> B terminar (1/10 sim, pois A *pode* terminar). Assim, a resposta seria 
>>> 1/10*1/10*2=1/50.
>>>
>>> Com mais detalhes para justificar o segundo "1/10":
>>> -- Número de sorteios possíveis = 10!
>>> -- Número de sorteios que formam um único ciclo de tamanho 10 = 9!
>>> -- Note que ter um ciclo de tamanho 10 equivale a terminar com quem inicia; 
>>> portanto, a chance de o amigo secreto terminar com o mesmo que iniciou 
>>> seria 9!/10!=1/10 (que é independente de quem começa).
>>>
>>> Assim:
>>> -- Chance de A iniciar = 1/10;
>>> Agora, DADO QUE A INICIOU:
>>>  Chance de A terminar = 9!/10! = 1/10
>>>  Portanto, chance de não terminar com A: 9/10
>>>  Chance de B terminar (por simetria): (9/10) /9 = 1/10
>>>
>>> Isso nos dá 1/10*1/10 = 1/100 de chance do amigo secreto começar por A e 
>>> terminar com B. Portanto a resposta seria o dobro, 1/50.
>>>
>>> ---///---
>>> SE AUTO-SORTEIOS SÃO PROIBIDOS:
>>> -- Número de sorteios (desarranjos) possíveis = !10 (vou escrever K=!10 
>>> daqui por diante);
>>> -- Número de sorteios que formam um único ciclo de tamanho 10 = 9!
>>> -- Portanto, a chance de o amigo secreto terminar com o mesmo que iniciou 
>>> seria 9!/K (que é independente de quem começa).
>>>
>>> Assim:
>>> -- Chance de A iniciar = 1/10;
>>> Agora, DADO QUE A INICIOU:
>>>  Chance de A terminar = 9!/K
>>>  Portanto, chance de não terminar com A: 1-9!/K
>>>  Chance de B terminar (por simetria): (1-9!/K) /9 = (K-9!)/(9K)
>>>
>>> Isso nos dá 1/10* (K-9!)/(9K) = (K-9!)/(90K) de chance do amigo secreto 
>>> começar por A e terminar com B. Portanto a resposta seria o dobro, 
>>> (K-9!)/(45K). Fazendo a conta com a ajuda do computador, achei 12001/741645.
>>>
>>> Abraço, Ralph.
>>>
>>>
>>>
>>> On Tue, Jan 26, 2021 at 1:45 PM Professor Vanderlei Nemitz 
>>>  wrote:

 Oi, pessoal!

 Com certeza vocês estão acompanhando desde domingo as resoluções da 
 questão do ENEM do amigo secreto.
 Além da resposta proposta, 1/45, que parece não estar correta, já vi 
 outras duas, 12001/741645 (ETAPA e ANGLO), que consideram também que o 
 sorteio anterior para definir "quem presenteia quem", e 7/360, do vídeo a 
 seguir:

 https://www.youtube.com/watch?v=c-t_BAMASKE

 Gostaria da opinião (e se possível, uma resolução) dos especialistas da 
 lista (Ralph e cia :))

 Muito obrigado!





-- 
Esta mensagem foi verificada pelo sistema de antiv�rus e
 acredita-se estar livre de perigo.


=
Instru��es para entrar na lista, sair da lista e usar a lista em
http://www.mat.puc-rio.br/~obmlistas/obm-l.html
=


[obm-l] Re: [obm-l] Re: [obm-l] Re: [obm-l] Re: [obm-l] Re: [obm-l] Re: [obm-l] Re: [obm-l] Re: Equações funcionais

2021-02-16 Por tôpico Anderson Torres
Em ter., 16 de fev. de 2021 às 21:26, joao pedro b menezes
 escreveu:
>
> Eu sei, temos f(-1)= 0, f(0) = 1, e f é bijetora. Após trabalhar a equação 
> que cheguei na expressão:
> f( x + f(x) ) - f( f(x)) = x.  Queria saber se essa identidade, junto com a 
> do enunciado, é suficiente para provar a linearidade de f.
>

Seriosamente, não me parece útil perder tempo provando que isso é
linear. O processo que você levaria provando que f(x)=Ax+B basicamente
se resumiria a finalizar o problema.

Outra identidade que pode ser útil para você é provar que f(f(x)) -
f(x) = f(x) -x.
Essa, junto com a identidade acima que você provou, te deixam em 70%
do problema.

> --
> Esta mensagem foi verificada pelo sistema de antivírus e
> acredita-se estar livre de perigo.

-- 
Esta mensagem foi verificada pelo sistema de antiv�rus e
 acredita-se estar livre de perigo.


=
Instru��es para entrar na lista, sair da lista e usar a lista em
http://www.mat.puc-rio.br/~obmlistas/obm-l.html
=


[obm-l] Re: [obm-l] Re: [obm-l] Re: [obm-l] Re: [obm-l] Re: [obm-l] Re: Equações funcionais

2021-02-16 Por tôpico Anderson Torres
Em ter., 16 de fev. de 2021 às 20:43, joao pedro b menezes
 escreveu:
>
> Foi da OBM 2006, nível 3,  3° fase:
> “Determine todas as funções f: R -> R tais que
> f( xf(y) + f(x) ) = 2f(x) + xy

Isso dá bem mais informação!

Por exemplo essa função é sobrejetora. Afinal, qualquer número pode
ser escrito na forma 2f(x)+xy - faça por exemplo x=1 e y=z-2f(1).

Daí a ideia é resolver as equacoes f(A)=0 e f(B)=1.

> para todos x,y reais”
>
> --
> Esta mensagem foi verificada pelo sistema de antivírus e
> acredita-se estar livre de perigo.

-- 
Esta mensagem foi verificada pelo sistema de antiv�rus e
 acredita-se estar livre de perigo.


=
Instru��es para entrar na lista, sair da lista e usar a lista em
http://www.mat.puc-rio.br/~obmlistas/obm-l.html
=


[obm-l] Re: [obm-l] Re: [obm-l] Re: [obm-l] Re: [obm-l] Sequência Injetiva

2021-02-16 Por tôpico Anderson Torres
Nada como uma bijeção N -> Q para encerrar o dia!

Se pensar nas operacoes INC e REV, podemos usar um algoritmo assim:

- Se o número é maior que 1, usa DEC (inversa de INC)
- Se o número é menor que 1, usa INV
- Se o número é 1, pare

Como demonstrar que este procedimento sempre encerrará em 1, não
importando que número racional começou? Acho que no fundo isso é só
uma maneira de encodar fracoes continuas mesmo.

Em ter., 16 de fev. de 2021 às 20:35, Matheus Secco
 escreveu:
>
> Esse problema caiu na Olimpíada Iberoamericana de 2009 que eu participei. Foi 
> o problema 5 da prova e lá pedia para provar injetividade e sobrejetividade.
>
> Em qua, 17 de fev de 2021 00:16, Anderson Torres 
>  escreveu:
>>
>> Em dom., 14 de fev. de 2021 às 17:20, Claudio Buffara
>>  escreveu:
>> >
>> > Será que essa sequência é sobrejetiva (sobre os racionais positivos)?
>> > Porque como a(2^n) = n+1, ela certamente atinge todos os naturais, de modo 
>> > que é ilimitada, superiormente e inferiormente (já que a(2^n + 1) = 
>> > 1/(n+1) ).
>> > Mesmo que não seja, seria interessante descobrir que racionais positivos 
>> > ela não atinge.
>> > É suficiente provar que todos os racionais entre 0 e 1 são atingidos (no 
>> > caso, pelos termos de ordem ímpar), mas não sei se isso facilita.
>> > Vale uma exploração numérica, talvez com uma planilha.
>>
>>
>> Se eu não errei as contas, acredito que sim. Afinal basta reverter a
>> fracao continua.
>>
>> As operacoes parecem ser bem limitadas, contudo nao e necessario muito
>> mais que isso para gerar um racional qualquer:
>>
>> - Função INC: x -> x+1
>> - Função REV: x -> 1/x
>>
>> Talvez haja algum invariante que permita prever que cada operacao esta
>> fadada a cair em 1
>>
>> >
>> >
>> > Abs,
>> > Claudio.
>> >
>> > Enviado do meu iPhone
>> >
>> > Em 14 de fev. de 2021, à(s) 13:57, Anderson Torres 
>> >  escreveu:
>> >
>> > 
>> >
>> >
>> > Em sáb., 13 de fev. de 2021 às 17:56, Jeferson Almir 
>> >  escreveu:
>> >>
>> >> Amigos, peço ajuda em provar a injetividade dessa sequência que seria 
>> >> uma saída para provar a unica ocorrência do racional que aparece nela. 
>> >> Estou andando em círculos tentando montar uma possível indução.
>> >>
>> >>
>> >> Dado a sequência a_1 = 1 e a_2n = a_n  + 1 e a_2n+1 = 1/a_2n.
>> >>
>> >> Prove que para todo racional positivo que ocorre na sequência, ocorre 
>> >> uma única vez.
>> >>
>> >>
>> >
>> > Acho que e uma boa usar fracao continua aqui.
>> >
>> > Se a_n = [c0; c1, c2, ..., ck], temos entao a_1 = [1] e
>> >
>> > a_2n =Â [(1+c0); c1, c2, ..., ck] (chamemos isso de operacao E)
>> > a_2n+1 = [0; (1+c0), c1, c2, ..., ck] (chamemos isso de operacao O)
>> >
>> >
>> > A partir disso, acredito que a bijecao fica quase obvia, bastando 
>> > formalizar algumas inducoes marotas.
>> >
>> > Primeiramente, nenhuma representacao da forma [...,N,1] vai surgir dai a 
>> > partir de a_2. Isso pode ser demonstrado por inducao mesmo: ck=1 somente 
>> > no caso [1], e depois dele a funcao a_n so modifica o comeco da cadeia, 
>> > nunca o final dela.
>> >
>> > Assim sendo, temos certeza que nao tem como um racional aparecer uma vez 
>> > na forma canonica e outra na forma alternativa. E, por conseguinte, se 
>> > duas fracoes tem comprimentos diferentes, elas devem ser diferentes. E 
>> > fracoes com comprimentos iguais diferem se e somente se pelo menos um dos 
>> > componentes diferir.
>> >
>> > Agora, a funcao recursiva age de duas formas. Uma delas altera o 
>> > comprimento em 1, e a outra mantém. A que altera, só altera 
>> > acrescentando o 0 na cabeceira. A que não altera, incrementa a cabeceira.
>> >
>> > Desta forma, é possível gerar de maneira unica qualquer numero racional 
>> > comecando do 1.
>> >
>> > - Qualquer fracao de comprimento 1 pode ser gerada simplesmente aplicando 
>> > a operacao E tantas vezes quantas forem necessarias. E tambem nao e 
>> > possivel fazer isso de outra maneira, pois a operacao O aumentara o 
>> > comprimento de maneira irreversivel.
>> >
>> > - Dada uma fracao com comprimento K, temos duas sub inducoes para fazer:
>> >
>> > + A fracao tem comprimento K e come

[obm-l] Re: [obm-l] Re: [obm-l] Re: [obm-l] Re: Equações funcionais

2021-02-16 Por tôpico Anderson Torres
Eu gostaria de saber da origem desse problema...

Em dom., 14 de fev. de 2021 às 14:32, joao pedro b menezes <
joaopedrobmene...@gmail.com> escreveu:

> Obrigado pela resposta, mas ainda tenho umas dúvidas. Poderia dar um
> exemplo de tal função ou explicar como construí-la? E se f fosse somente
> injetora, mudaria alguma coisa?
>
>
> --
> Esta mensagem foi verificada pelo sistema de antivírus e
> acredita-se estar livre de perigo.

-- 
Esta mensagem foi verificada pelo sistema de antiv�rus e
 acredita-se estar livre de perigo.



[obm-l] Re: [obm-l] Re: [obm-l] Sequência Injetiva

2021-02-16 Por tôpico Anderson Torres
Em dom., 14 de fev. de 2021 às 17:20, Claudio Buffara
 escreveu:
>
> Será que essa sequência é sobrejetiva (sobre os racionais positivos)?
> Porque como a(2^n) = n+1, ela certamente atinge todos os naturais, de modo 
> que é ilimitada, superiormente e inferiormente (já que a(2^n + 1) = 1/(n+1) ).
> Mesmo que não seja, seria interessante descobrir que racionais positivos ela 
> não atinge.
> É suficiente provar que todos os racionais entre 0 e 1 são atingidos (no 
> caso, pelos termos de ordem ímpar), mas não sei se isso facilita.
> Vale uma exploração numérica, talvez com uma planilha.


Se eu não errei as contas, acredito que sim. Afinal basta reverter a
fracao continua.

As operacoes parecem ser bem limitadas, contudo nao e necessario muito
mais que isso para gerar um racional qualquer:

- Função INC: x -> x+1
- Função REV: x -> 1/x

Talvez haja algum invariante que permita prever que cada operacao esta
fadada a cair em 1

>
>
> Abs,
> Claudio.
>
> Enviado do meu iPhone
>
> Em 14 de fev. de 2021, à(s) 13:57, Anderson Torres 
>  escreveu:
>
> 
>
>
> Em sáb., 13 de fev. de 2021 às 17:56, Jeferson Almir 
>  escreveu:
>>
>> Amigos, peço ajuda em provar a injetividade dessa sequência que seria uma 
>> saída para provar a unica ocorrência do racional que aparece nela. Estou 
>> andando em círculos tentando montar uma possível indução.
>>
>>
>> Dado a sequência a_1 = 1 e a_2n = a_n  + 1 e a_2n+1 = 1/a_2n.
>>
>> Prove que para todo racional positivo que ocorre na sequência, ocorre uma 
>> única vez.
>>
>>
>
> Acho que e uma boa usar fracao continua aqui.
>
> Se a_n = [c0; c1, c2, ..., ck], temos entao a_1 = [1] e
>
> a_2n =Â [(1+c0); c1, c2, ..., ck] (chamemos isso de operacao E)
> a_2n+1 = [0; (1+c0), c1, c2, ..., ck] (chamemos isso de operacao O)
>
>
> A partir disso, acredito que a bijecao fica quase obvia, bastando formalizar 
> algumas inducoes marotas.
>
> Primeiramente, nenhuma representacao da forma [...,N,1] vai surgir dai a 
> partir de a_2. Isso pode ser demonstrado por inducao mesmo: ck=1 somente no 
> caso [1], e depois dele a funcao a_n so modifica o comeco da cadeia, nunca o 
> final dela.
>
> Assim sendo, temos certeza que nao tem como um racional aparecer uma vez na 
> forma canonica e outra na forma alternativa. E, por conseguinte, se duas 
> fracoes tem comprimentos diferentes, elas devem ser diferentes. E fracoes com 
> comprimentos iguais diferem se e somente se pelo menos um dos componentes 
> diferir.
>
> Agora, a funcao recursiva age de duas formas. Uma delas altera o comprimento 
> em 1, e a outra mantém. A que altera, só altera acrescentando o 0 na 
> cabeceira. A que não altera, incrementa a cabeceira.
>
> Desta forma, é possível gerar de maneira unica qualquer numero racional 
> comecando do 1.
>
> - Qualquer fracao de comprimento 1 pode ser gerada simplesmente aplicando a 
> operacao E tantas vezes quantas forem necessarias. E tambem nao e possivel 
> fazer isso de outra maneira, pois a operacao O aumentara o comprimento de 
> maneira irreversivel.
>
> - Dada uma fracao com comprimento K, temos duas sub inducoes para fazer:
>
> + A fracao tem comprimento K e comeca com 0.
>
> Â  Entao ela foi gerada por uma operacao O. O elemento que a gerou tinha 
> menos componentes, os quais satisfazem a hipotese de inducao.
>
> + A fracao tem comprimento K e comeca com algo maior que 0.
>
> Entao ela foi gerada por uma operacao E. A fracao da qual ela foi gerada 
> difere unicamente no primeiro elemento, o qual antes era menor. Assim sendo, 
> e possivel reduzir isso ate chegar no caso anterior.
>
> E isso demonstra recursivamente a unicidade e existencia!
>
>
>
>> --
>> Esta mensagem foi verificada pelo sistema de antivírus e
>> acredita-se estar livre de perigo.
>
>
> --
> Esta mensagem foi verificada pelo sistema de antivírus e
> acredita-se estar livre de perigo.
>
>
> --
> Esta mensagem foi verificada pelo sistema de antivírus e
> acredita-se estar livre de perigo.

-- 
Esta mensagem foi verificada pelo sistema de antiv�rus e
 acredita-se estar livre de perigo.


=
Instru��es para entrar na lista, sair da lista e usar a lista em
http://www.mat.puc-rio.br/~obmlistas/obm-l.html
=


[obm-l] Re: [obm-l] Re: Equações funcionais

2021-02-14 Por tôpico Anderson Torres
Em dom., 14 de fev. de 2021 às 11:30, joao pedro b menezes <
joaopedrobmene...@gmail.com> escreveu:

> Obs: f é bijetora
>
>>
>
Acho que nao basta. Se f(x)=y entao f(x+y)=x+f(y).

Com isso, poderiamos fazer uma funcao que nao aja linearmente em (0,1) mas
aja linearmente fora dele.



> --
> Esta mensagem foi verificada pelo sistema de antivírus e
> acredita-se estar livre de perigo.

-- 
Esta mensagem foi verificada pelo sistema de antiv�rus e
 acredita-se estar livre de perigo.



[obm-l] Re: [obm-l] Sequência Injetiva

2021-02-14 Por tôpico Anderson Torres
Em sáb., 13 de fev. de 2021 às 17:56, Jeferson Almir <
jefersonram...@gmail.com> escreveu:

> Amigos, peço ajuda em provar a injetividade dessa sequência que seria uma
> saída para provar a unica ocorrência do racional que aparece nela. Estou
> andando em círculos tentando montar uma possível indução.
>
>
> Dado a sequência a_1 = 1 e a_2n = a_n  + 1 e a_2n+1 = 1/a_2n.
>
> Prove que para todo racional positivo que ocorre na sequência, ocorre uma
> única vez.
>
>
Acho que e uma boa usar fracao continua aqui.

Se a_n = [c0; c1, c2, ..., ck], temos entao a_1 = [1] e

a_2n = [(1+c0); c1, c2, ..., ck] (chamemos isso de operacao E)
a_2n+1 = [0; (1+c0), c1, c2, ..., ck] (chamemos isso de operacao O)


A partir disso, acredito que a bijecao fica quase obvia, bastando
formalizar algumas inducoes marotas.

Primeiramente, nenhuma representacao da forma [...,N,1] vai surgir dai a
partir de a_2. Isso pode ser demonstrado por inducao mesmo: ck=1 somente no
caso [1], e depois dele a funcao a_n so modifica o comeco da cadeia, nunca
o final dela.

Assim sendo, temos certeza que nao tem como um racional aparecer uma vez na
forma canonica e outra na forma alternativa. E, por conseguinte, se duas
fracoes tem comprimentos diferentes, elas devem ser diferentes. E fracoes
com comprimentos iguais diferem se e somente se pelo menos um dos
componentes diferir.

Agora, a funcao recursiva age de duas formas. Uma delas altera o
comprimento em 1, e a outra mantém. A que altera, só altera acrescentando o
0 na cabeceira. A que não altera, incrementa a cabeceira.

Desta forma, é possível gerar de maneira unica qualquer numero racional
comecando do 1.

- Qualquer fracao de comprimento 1 pode ser gerada simplesmente aplicando a
operacao E tantas vezes quantas forem necessarias. E tambem nao e possivel
fazer isso de outra maneira, pois a operacao O aumentara o comprimento de
maneira irreversivel.

- Dada uma fracao com comprimento K, temos duas sub inducoes para fazer:

+ A fracao tem comprimento K e comeca com 0.

  Entao ela foi gerada por uma operacao O. O elemento que a gerou tinha
menos componentes, os quais satisfazem a hipotese de inducao.

+ A fracao tem comprimento K e comeca com algo maior que 0.

Entao ela foi gerada por uma operacao E. A fracao da qual ela foi gerada
difere unicamente no primeiro elemento, o qual antes era menor. Assim
sendo, e possivel reduzir isso ate chegar no caso anterior.

E isso demonstra recursivamente a unicidade e existencia!



-- 
> Esta mensagem foi verificada pelo sistema de antivírus e
> acredita-se estar livre de perigo.

-- 
Esta mensagem foi verificada pelo sistema de antiv�rus e
 acredita-se estar livre de perigo.



[obm-l] Re: [obm-l] Re: [obm-l] Indução

2021-02-09 Por tôpico Anderson Torres
Disfarce o Lema da Boa Ordenacao, dado que e equivalente ao principio da
inducao.

Em sex., 5 de fev. de 2021 às 07:31, joao pedro b menezes <
joaopedrobmene...@gmail.com> escreveu:

> obs: só agora fui ver o título :) , se era necessário fazer especialmente
> por indução, por favor desconsidere a minha resposta.
>
> On Fri, Feb 5, 2021 at 7:14 AM joao pedro b menezes <
> joaopedrobmene...@gmail.com> wrote:
>
>> Suponha que d | (a^(2)^n ) + 1. Então a^2^n = -1 (mod d). Pegue um primo
>> tal que p| d, então a^2^n = -1 (mod p). Mas temos: a^2^(n+1) = 1 (mod p).
>> Logo
>>  ord(p)a | 2^(n+1), mas ord(p)a não divide 2^n, logo ord(p)a = 2^(n + 1).
>> Isso é um absurdo, pois ord(p)a < p <= d <= 2^(n + 1).
>> obs: tenho quase certeza que já perguntaram a mesma coisa nessa lista.
>> Portanto acho que vale a pena ir procurar a resposta anterior também :)
>>
>> On Thu, Feb 4, 2021 at 11:20 PM Heitor Gama Ribeiro <
>> heitor...@hotmail.com> wrote:
>>
>>> Prove por indução que se 3<= d <= 2^(n+1), então d não divide
>>> [a^(2)^(n) + 1] para todo inteiro positivo a.
>>>
>>>
>>> Sent from my iPhone
>>>
>>> =
>>> Instruções para entrar na lista, sair da lista e usar a lista em
>>> http://www.mat.puc-rio.br/~obmlistas/obm-l.html
>>> =
>>>
>>

-- 
Esta mensagem foi verificada pelo sistema de antiv�rus e
 acredita-se estar livre de perigo.



Re: [obm-l] PIF

2021-01-22 Por tôpico Anderson Torres
Em ter., 19 de jan. de 2021 às 21:25, Phablo dos Santos <
phablodosan...@gmail.com> escreveu:

> Prove que se 3<= d <= 2^(n+1), entao d nao divide [a^(2)^(n) + 1]. Para
> todo inteiro positivo a.
>
>
Seja p>2 um fator primo de a^(2^n)+1. Assim, MDC(p,a)=1 (isso deveria ser
óbvio), e portanto pelo pequeno Fermat, p é fator primo de a^p-1.
Mas p também é fator primo de a^(2^(n+1))-1, o que implica que p é fator
primo de a^g-1 onde g=MDC(2^(n+1), p-1).
Note que g não pode ser 2^n, senão p seria fator primo de a^(2^n)-1, e
portanto fator de 2, falso pois p é maior que 2.
Assim, g=2^(n+1), e portanto 2^(n+1) é divisor de p-1. Isso acarreta
p>=2^(n+1)+1.


[obm-l] Re: [obm-l] Raízes racionais

2021-01-16 Por tôpico Anderson Torres
Em ter., 12 de jan. de 2021 às 06:59, marcone augusto araújo borges <
marconeborge...@hotmail.com> escreveu:

> A equação ax^2 + bx + c = 0, com a, b e c inteiros tem duas raízes
> racionais cuja soma é igual ao produto. Qual a relação entre os
> coeficientes a e c?
>

As raízes são da forma p/q, p/(p-q) com p e q inteiros.

Assim, q(p-q)x^2-p^2x+p^2=0.

Assim, a=q(p-q) e b=p^2.


[obm-l] Re: [obm-l] Re: [obm-l] Ângulos de um triângulo

2020-12-06 Por tôpico Anderson Torres
Em sáb., 5 de dez. de 2020 às 07:15, Julio César Saldaña Pumarica <
saldana...@pucp.edu.pe> escreveu:

> É verdade, 30 graus é o DAB, más a pergunta era DAC
>
> o  DAC=18
>
>
> On Fri, Dec 4, 2020, 19:23 Julio César Saldaña Pumarica <
> saldana...@pucp.edu.pe> wrote:
>
>> Tenho uma solução com traços auxiliares. Resposta: 30°
>>
>> Tem como passar uma foto nesta lista?
>>
>

Use um site de compartilhamento de fotos, como o IMGUR.

Ou faça melhor: descreva a figura como se fosse explicar para um cego.



>
>> On Mon, Nov 30, 2020, 19:42 Professor Vanderlei Nemitz <
>> vanderma...@gmail.com> wrote:
>>
>>> Boa noite!
>>> Alguém conhece uma saída para o seguinte problema?
>>> Muito obrigado!
>>>
>>> *Num triângulo isósceles ABC, AB = AC.*
>>> *Seja D um ponto interno tal que os ângulos DBC, DCB, DBA e DCA medem,
>>> respectivamente, 12°, 18°, 54° e 48°. *
>>> *Determine a medida do ângulo DAC.*
>>>
>>>
>>> 
>>>  Livre
>>> de vírus. www.avast.com
>>> .
>>>
>>> <#m_-3450317920574181893_m_-1377089701738023066_m_6233189016624438778_m_1892860381202400781_DAB4FAD8-2DD7-40BB-A1B8-4E2AA1F9FDF2>
>>>
>>


[obm-l] Re: [obm-l] Ângulos de um triângulo

2020-12-03 Por tôpico Anderson Torres
Em seg., 30 de nov. de 2020 às 19:28, Professor Vanderlei Nemitz <
vanderma...@gmail.com> escreveu:

> Boa noite!
> Alguém conhece uma saída para o seguinte problema?
> Muito obrigado!
>
> *Num triângulo isósceles ABC, AB = AC.*
> *Seja D um ponto interno tal que os ângulos DBC, DCB, DBA e DCA medem,
> respectivamente, 12°, 18°, 54° e 48°. *
> *Determine a medida do ângulo DAC.*
>

Eu ainda nao resolvi, mas sei que e 30 graus.



>
>
> 
>  Livre
> de vírus. www.avast.com
> .
> <#m_-7214013898439594478_DAB4FAD8-2DD7-40BB-A1B8-4E2AA1F9FDF2>
>


[obm-l] Re: [obm-l] Re: [obm-l] Re: [obm-l] Re: [obm-l] Relação de girard

2020-11-12 Por tôpico Anderson Torres
Em sáb., 7 de nov. de 2020 às 16:44, Israel Meireles Chrisostomo <
israelmchrisost...@gmail.com> escreveu:

>   o objetivo dessa proposta é recriar o ambiente vivido por Euler na
> época.
>

E naquele tempo eles não usavam indução? Formalização é algo bem recente na
matemática.

Sua exigência me parece algo tão surreal quanto exigir rigor na geometria
do tempo de Euclides.



>
> Em sáb., 7 de nov. de 2020 às 15:10, Israel Meireles Chrisostomo <
> israelmchrisost...@gmail.com> escreveu:
>
>> Na verdade eu estava elaborando um problema que dependia disso.O problema
>> é esse aqui:
>>
>> Desafio do ano: resolver o problema da Basiléia sem usar derivadas,
>> integrais, série de potências, produto infinito do seno ou cosseno, ou
>> mesmo indução ou números complexos.
>>
>> Em sáb., 7 de nov. de 2020 às 15:07, Israel Meireles Chrisostomo <
>> israelmchrisost...@gmail.com> escreveu:
>>
>>> Na verdade eu estava elaborando um problema que dependia disso.O
>>> problema é esse aqui:
>>>
>>> Desafio do ano: resolver o problema da Basiléia sem usar derivadas,
>>> integrais, série de potências, produto infinito do seno ou cosseno, ou
>>> mesmo indução.
>>>
>>> Em sáb., 7 de nov. de 2020 às 14:47, Israel Meireles Chrisostomo <
>>> israelmchrisost...@gmail.com> escreveu:
>>>
 conheço uma que usa o teorema de d'lambert

 Em sáb., 7 de nov. de 2020 às 12:50, Bernardo Freitas Paulo da Costa <
 bernardo...@gmail.com> escreveu:

> On Thu, Nov 5, 2020 at 9:26 PM Artur Costa Steiner
>  wrote:
> >
> > Para facilitar, suponhamos que o polinômio de grau n P seja mônico.
> Sejam z_1, , z_n suas n raízes não necessariamente distintas. Para 
> todo
> complexo z, temos que
> >
> > P(z)  = ( z - z_1) (z - z_n)
> >
> > Desenvolvendo e aplicando o chamado produto de Stevin, vc tem as
> relações de Girard.
>
> Eu não conhecia o produto de Stevin, mas de forma geral quando você
> usa "..." tem, muitas vezes, um argumento por indução que está
> subentendido.  Pode ser que o produto de Stevin "faça a indução pra
> você" (calculando os termos \sum \prod z_i que vão aparecer como
> coeficientes dos monômios z^k), mas é "quase" como se você estivesse
> empurrando a indução um andar abaixo ;-)
>
>
> Israel: qual a demonstração por indução que você conhece?  E porque
> você gostaria de outra??
>
> Abraços,
> --
> Bernardo Freitas Paulo da Costa
>
>
> =
> Instru�ões para entrar na lista, sair da lista e usar a lista em
> http://www.mat.puc-rio.br/~obmlistas/obm-l.html
>
> =
>


 --
 Israel Meireles Chrisostomo

>>>
>>>
>>> --
>>> Israel Meireles Chrisostomo
>>>
>>
>>
>> --
>> Israel Meireles Chrisostomo
>>
>
>
> --
> Israel Meireles Chrisostomo
>


Re: [obm-l] Probabilidade - duas listas a partir da normal(0,1)

2020-11-12 Por tôpico Anderson Torres
Não consigo ver nada

Em qua., 11 de nov. de 2020 às 14:52, Pedro Lazéra 
escreveu:

>


Re: [obm-l] Prova interessante de que lim n ---> oo n^(1/n) = 1

2020-11-01 Por tôpico Anderson Torres
Em qua., 28 de out. de 2020 às 08:03, Artur Costa Steiner <
artur.costa.stei...@gmail.com> escreveu:

> Achei essa prova bem imaginativa.
>

Eu acho que provar que log(n)/n tende a 0 quando n tende a infinito é
conceitualmente mais interessante.

Ou que e^n/n tende a infinito.



>
> Para n>= 2, temos n^(1/n) > 1. n^(1/n) pode ser escrito como
>
> n^(1/n) = ((raiz(n) . raiz(n) . 1  1)^(1/n)
>
>  onde o 1 aparece n - 2 vezes. Logo, n^(1/n) é a média geométrica dos
> números {raiz(n), raiz(n), 1, . .1}.
>
> Pela desigualdade MA >= MG  temos, para n>= 2, que
>
> 1 < n^(1/n) < (raiz(n) + raiz(n) + 1 +1)/n= (2 raiz(n) + (n - 2))/n
>
> 1 < n^(1/n) <  2/raiz(n) + 1 - 2/n
>
> Como na desigualdade acima o membro da direita tende a 1 quando n vai para
> oo, segue-se por confronto que
>
> lim n ---> oo n^(1/n) = 1
>
> Artur
>


[obm-l] Re: [obm-l] Teorema Chinês do resto

2020-10-29 Por tôpico Anderson Torres
Em ter., 27 de out. de 2020 às 20:50, joao pedro b menezes <
joaopedrobmene...@gmail.com> escreveu:

> Olá, eu estava fazendo esse exercício :
> " . (OBM 2005) Dados os inteiros positivos a, c e o inteiro b, prove que
> existe um inteiro positivo x tal que a^x + x ≡ b (mod c)."
>
> Eu pensei nessa solução, mas eu tenho quase certeza que ela está errada...
>
> "Primeiramente , suponhamos c primo. Desse modo, se escolhermos x tal que
> x  ≡ 0 (mod c - 1) , teremos a^x + x  ≡ 1 + x <=> x  ≡ b - 1 (mod c) (pelo
> teorema de fermat) . Teríamos o sistema de congruências:
> x  ≡ 0 ( mod c)
> x  ≡ b - 1 (mod c-1)
> Como c e c-1 são primos entre sí, pelo teorema chinês do resto esse
> sistema infinitas soluções.
> Agora, suponhamos c composto. Como um número composto é nada mais que o
> produto de uma quantidade finita de primos, podemos chamar todos os primos
> divisores de c como p1, p2, p3 ... pn . Forçando  x  ≡ b - 1 (mod pi) para
> qualquer pi divisor primo de c, montamos o sistema de congruências:
> x  ≡ 0 (mod p1 - 1)
> x  ≡ b - 1 (mod p1)
> .
> x  ≡ 0 (mod pn - 1)
> x  ≡ b - 1 (mod pn)
>

Apenas olhando por cima, você não pode ignorar os expoentes da fatoração.

Além disso, Euler-Fermat exige que a seja primo com c.


> O único empecilho para o teorema é que pj - 1 e ph - 1 ( com j e h
> inteiros 1 <= j <= h <= n) possivelmente terão múltiplos em comum. Para
> anular esse problema, basta fazer com que x seja múltiplo de p1 - 1, p2 - 1
> . pn - 1,e chamando de Z o produto de todos esses números, podemos
> construir:
> x  ≡ b - 1 (mod p1)
> x  ≡ b - 1( mod p2)
> 
> x  ≡ b - 1 (mod pn)
> x  ≡ 0 (mod Z)
> Como p1, p2 , ... pn e  Z são primos entre si, o sistema sempre terá
> infinitas soluções pelo teorema chinês do resto
> Dessa forma, comprovamos o enunciado"
>
> Se ela estiver errada( o que eu tenho quase certeza) , alguém poderia, por
> favor, me falar por que ?
> Agradeço pela ajuda e pelo tempo por ler este email gigante 
>


Re: [obm-l] Lista/ Livros Geometria IMO/OBM

2020-10-27 Por tôpico Anderson Torres
Eu compilei umas listas faz um bom tempo no Bitbucket. Pretendo mudar o
repositório no futuro, mas até lá divirta-se:

https://bitbucket.org/anderson_torres/junkyard/src/master/

Em seg., 26 de out. de 2020 às 20:48, Jones Colombo 
escreveu:

> Oi RF -romelsfmath, um lugar para você aprender um porção de coisas é
> olhar os arquivos desta lista de problemas
> http://www.mat.puc-rio.br/~obmlistas/obm-l.html lá você vai encontrar
> muito material para estudar.
> [@]
> Jones
>
> On Sun, Oct 25, 2020 at 1:08 PM joao pedro b menezes <
> joaopedrobmene...@gmail.com> wrote:
>
>> Olá, boa tarde. Eu não conheço todos, mas eu sei que é possivel entrar no
>> site da OBM :
>>
>> https://www.obm.org.br/2020/07/25/conheca-livros-para-iniciar-a-preparacao-para-a-proxima-obm/
>>
>> Ainda assim, um livro que eu particularmente acho fantástico se chama
>> “Challenging  problems in geometry “. Ele é usado para a preparação da IMO.
>>
>


[obm-l] Re: [obm-l] Re: [obm-l] Animação do site da OBM

2020-10-19 Por tôpico Anderson Torres
O algoritmo de animação não está exatamente disponível, mas o artigo da OBM
sobre o Porisma de Steiner explica bem a sua ideia: invertendo um par de
círculos concêntricos, é possível produzir qualquer configuração de Steiner.

Em sáb., 17 de out. de 2020 às 15:41, Leonardo Borges Avelino <
lbor...@gmail.com> escreveu:

> Trata-se do tema de inversão e tem um artigo na Eureka 4
> https://www.obm.org.br/content/uploads/2017/01/eureka4.pdf
>
> Abs
>
> On Sat, Oct 17, 2020 at 3:14 PM Felippe Coulbert Balbi <
> felippeba...@hotmail.com> wrote:
>
>> A muitos anos atras durante um coloquio de matemática no IMPA, no grupo
>> de olimpíada, estavam resolvendo um problema se não me engano da IMO e
>> durante a resolução houve um comentário dessa resolução com a animação do
>> logo no site da OBM.
>>
>> Eu estou fazendo um programa que tem haver com vídeos e queria colocar
>> essa animação no meu programa. Eu procurei meus materiais antigos sobre o
>> algoritmo para gerar ele mas não achei nada.
>>
>> Alguém consegue dar uma força?
>>
>> Até,
>> Felippe
>>
>>
>> 
>>  Livre
>> de vírus. www.avg.com
>> .
>>
>> <#m_-2163421921828132999_m_-8807507600612609615_DAB4FAD8-2DD7-40BB-A1B8-4E2AA1F9FDF2>
>>
>


Re: [obm-l] Problema da IMO

2020-09-22 Por tôpico Anderson Torres
Em sáb., 12 de set. de 2020 às 01:18, Pedro José  escreveu:
>
> Boa noite!
> Atrapalhou meu vinho e o filme que estava assistindo mas consegui. Não gostei 
> tanto, agora que consegui, é muito trabalhoso.
>
> 2= [3(y+1)(z+1)-1]/2yz
> yz= 3(yz+2) (i)
> z(y-3)= 3y +2 (ii)
> y(z-3)=3z+2 (iii)
> (i)*(ii) yz(z-3)(y-3)= 9yz+6(y+z)+4 e Voilá: (z-3)(y-3)=11.
>
> Saudações,
> PJMS
>
>
>
>
>
> Em sáb., 12 de set. de 2020 às 00:35, Pedro José  
> escreveu:
>>
>> Boa noite!
>> Fui em uma linha parecida com a primeira solução, embora não visse 
>> necessidade de mudança de variáveis.
>> Mas o b achei sempre por restrição.
>> Esse "it implies" e aparece um número fatorado, não consegui captar, embora 
>> tenha gostado do recurso, já que é bem restritivo.
>>
>> Sudações,
>> PJMS
>>
>>
>> Em sáb., 12 de set. de 2020 às 00:08, Pedro José  
>> escreveu:
>>>
>>> Boa noite!
>>> Grato, Ralph!
>>>
>>> Estou estudando a solução. Pelo menos, não me decepcionei. A resposta 
>>> estava correta,
>>>
>>> Saudações.
>>> PJMS
>>>
>>> Em sex., 11 de set. de 2020 às 22:33, Ralph Costa Teixeira 
>>>  escreveu:

 Essa eh da IMO 1992. Tem uma solucao aqui:
 http://sms.math.nus.edu.sg/Simo/IMO_Problems/92.pdf

 On Fri, Sep 11, 2020 at 10:06 PM Pedro José  wrote:
>
> Bom dia!
>
> Recebi de um filho de um amigo, um problema que já o fizera.
> (a-1)(b-1)(c-1) | abc-1;  1
> Confesso que desta feita gastei mais tempo que da  primeira vez. Curioso, 
> da primeira ,eu pensei, dessa vez, eu tentei lembrar como eu resolvera, 
> aí nem lembrava, nem pensava. Apelei para a internet, mas não encontrei 
> nada. Mas no fim, recordei o que havia feito.
> (1+1/(a-1))(1+1/(b-1))(1+1/(c-1)) = k, onde k é inteiro.
> vê-se que k>1, e para um dado a k é máximo para b e c mínimos logo b=a+1 
> e c=a+2
> [a(a+1)(a+2)]/[(a-1)(a)(a+1)] > [a(a+1)(a+2)-1]/[(a-1)(a)(a+1)]>=2, então 
> (a+2)/(a-1)>2 ==> a <4, a=2 ou a=3.
> O k é máximo para a=2, b=3 e c=4 ==> k <4, logo k=2 ou k=3.
> S.p.g, se a é ímpar (a-1)(b-1)(c-1) é par; então b,c ímpares e k é livre.
> S.p.g se a é par abc-1 é ímpar; então b,c são pares e k ímpar.
> a=2, temos  2b(b+1)/[(b-1)b] >3, não usei a restrição de paridade para c 
> para facilitar a simplificação. b<5 Logo a=2 k é ímpar k=3. Logo c= 8. (2,4,8) é uma solução.
>
> a=3 temos 3b(b+1)/[2(b-1)b] > 2; b<7 e 3 para a=3 e b=5. kmax <= (15*7-1)/(2*4*6) <=2;pois k é inteiro.
> 1 k=2 e c= 15. (3,5,15) é a outra solução.
>
> Só agora me apercebi de que c=ab nas duas soluções. Então tentei uma nova 
> solução.
> (a-1)(b-1)(c-1) | abc-1 e (a-1)(b-1)(c-1) | abc + c(1 - (a+b)) -ab+ (a+b) 
> - 1 logo divide a diferença:
> (a-1)(b-1)(c-1) | (a+b) (c-1) + ab -1 - (c-1) logo c-1 | ab-1, então 
> ab-1= w(c-1), para algum w inteiro e ab=w(c-1) +1 (i)
> Como a=2 ou a=3
> Se a=2. e w>=2
> Temos por (i) 2b>= 2 (c-1) +1 c-1>=b, logo absurdo.
> Se a=3
> Temos por (i) 3b>= w(c-1)+1; w=3 ==>3b< 3 (c-1) +1 pois c>b
> w=2 ==> 3b =2(c-1) +1 ==> c=(3b+1)/2
> 2(b-1)(3b-1)/2 | 3b(3b+1)/2 -1 ==>  2(b-1)(3b-1)/ | 3b(3b+1) -2 ==> 
> 6b^2-8b-2 | 9b^2+3b-1 ==>  6b^2-8b-2 | 3b^2 +11b+ 4
> ==> b <=5. Como b>a=3 ==> b=5 e c= 8, ferindo a paridade.
> Logo ab-1=c-1 ==> ab=c ==> (a-1)(b-1)(c-1) | c^2-1 ==>  (a-1)(b-1) |  c+1 
> (a-1)(b-1) |ab+1==> (a-1)(b-1)!a+b
> a=2 ==> 2b-2= 2+b. b=4 e c=ab=8 (2,4,8)
> a=3 ==> 2(b-1) | 3+b ==> 2(b-1) = 3+. b=5 e c=ab=15. (3,5,15),
> Forcei um pouco a barra para mostrar que c=ab.
> Alguém teria uma outra solução, ou um endereço onde se tem as questões da 
> IMO e suas resoluções?

O bom e velho MathLinks! Ou melhor, AOPS:

https://artofproblemsolving.com/community/c89

A bem da verdade, eu tinha pego minha antiga papelada e convertido uma
imensa parte para LaTeX. Entre estas, estavam várias listas de
exercícios, além de problemas das IMOs. Em breve vou jogar no
Bitbucket.

>
> Grato!
> Saudações,
> PJMS
>
>
>
> --
> Esta mensagem foi verificada pelo sistema de antivírus e
> acredita-se estar livre de perigo.


 --
 Esta mensagem foi verificada pelo sistema de antivírus e
 acredita-se estar livre de perigo.
>
>
> --
> Esta mensagem foi verificada pelo sistema de antivírus e
> acredita-se estar livre de perigo.

-- 
Esta mensagem foi verificada pelo sistema de antiv�rus e
 acredita-se estar livre de perigo.


=
Instru��es para entrar na lista, sair da lista e usar a lista em
http://www.mat.puc-rio.br/~obmlistas/obm-l.html
=


[obm-l] Re: [obm-l] Re: [obm-l] Re: [obm-l] Re: [obm-l] Re: [obm-l] Re: [obm-l] Geometria plana com desigualdade de médias?

2020-08-27 Por tôpico Anderson Torres
Em qua., 26 de ago. de 2020 às 18:29, Pedro José  escreveu:
>
> Boa noite!
> Anderson,
> achei legal a sua visão. Mas não consegui evoluir com nada.
> Todavia, fiquei com uma dúvida. Como x+y é um dos ângulos do triângulo temos 
> a restrição 0 E entendo que tanto para cotg(x) + cot(y) , como para tg(x) + tg(y) ocorrerá 
> um mínimo em x=y=K/2, onde x+y=k,k sendo um constante.
> Não acompanhei a sua dedução d quando um é mínimo o outro é máximo.

Eu não fui muito claro.

Você converteu o problema em "calcule o valor mínimo de cot(x)+cot(y)
com x+y fixo". Isso é essencialmente o mesmo que resolver o problema
"calcule o valor mínimo de tan(a)+tan(b) com a+b fixo" - pois sabendo
resolver um é só usar a mesma solução para x=90-a, y=90-b.

>
> Saudações,
> PJMS
>
> Em qui., 20 de ago. de 2020 às 22:40, Anderson Torres 
>  escreveu:
>>
>> Em qui., 20 de ago. de 2020 às 22:03, Anderson Torres
>>  escreveu:
>> >
>> > Em ter., 18 de ago. de 2020 às 19:51, Pedro José  
>> > escreveu:
>> > >
>> > > Boa noite!
>> > > Cláudio,
>> > > não consegui nada geométrico.
>> > > O máximo que atingi foi:
>> > > a/ha + b/hb + c/hc= [cotg(A1) +cotg (A2)]  + [cotg(B1) +cotg (B2)] + 
>> > > co[tg(C1) +cotg (C2)] com A1 + A2 = A; B1 + B2 + B e C1 + C2 = C.
>> > > Para ser mínimo cada termo entre colchetes deve ser mínimo, o que ocorre 
>> > > quando A1 = A2; B1 = B2 e C1 = C2. Logo P seria o encontro das 
>> > > bissetrizes e logo I.
>> > > Onde: A1= PAB e A2=PAC; B1=PBA e B2=PBC; C1=PCA e C2=PCB.
>> >
>> > Acho que daqui poderia sair uma interpretação mais escamoteada.
>> > Afinal, trigonometria é uma espécie de "ponto de contato" entre a
>> > geometria analítica e a sintética, entre a nuvem de desenhos e a de
>> > números.
>> >
>> > Acredito que a solução aqui seria arranjar uma interpretação
>> > geométrica desses colchetes de co-tangentes. Acredito que possamos
>> > apelar para Ptolomeu em algum momento ou para um macete de
>> > semelhanças, pois as projeções de um ponto sobre duas retas criam um
>> > quadrilátero cíclico.
>>
>> Acrescentando mais coisas: se queremos minimizar cot(x) +cot(y) com
>> x+y fixo, isto é equivalente a minimizar tan(90-x)+tan(90-y) com
>> 90-x+90-y fixo. Ou como maximizar tan(x) + tan(y) com x+y fixo.
>>
>> Geometricamente, tangente é cateto oposto dividido por cateto
>> adjacente. Logo uma soma de tangentes com catetos adjacentes iguais
>> equivale a uma soma de catetos opostos! Assim sendo, nosso problema
>> pode ser pensado da seguinte forma:
>>
>> Dados um ponto A e uma reta d fixos, temos que construir duas retas x
>> e y, com ângulo 'alfa' entre elas, ambas passando por A e tais que a
>> distância entre os pontos X e Y, que elas geram ao intersectar d, seja
>> mínima.
>>
>> Daí fica fácil argumentar que a altura por A também tem que ser a
>> bissetriz por A.
>>
>> No fundo do fundo é uma forma de geometrizar a solução trigonométrica.
>> A trigonometria se torna apenas um atalho.
>>
>> Vou formalizar isso mais tarde, com desenhos e tudo.
>>
>>
>>
>> >
>> > Isso até me lembra o famoso artigo do Shine sobre geometria cearense
>> > VS geometria paulista:
>> > https://cyshine.webs.com/geometria-2005.pdf
>> >
>> >
>> > >
>> > > Saudações,
>> > > PJMS
>> > >
>> > > Em ter., 18 de ago. de 2020 às 11:34, Claudio Buffara 
>> > >  escreveu:
>> > >>
>> > >> Será que tem uma demonstração mais geométrica e menos algébrica disso? 
>> > >> E que torne o resultado mais intuitivo?
>> > >> É razoável que o ponto P não esteja muito próximo de qualquer dos 
>> > >> lados, pois neste caso, se P se aproximasse do lado a, por exemplo, 
>> > >> a/h_a cresceria e a expressão se afastaria do valor mínimo.
>> > >> Mas, com lados não necessariamente congruentes, não é óbvio, a priori, 
>> > >> que P deva ser equidistante dos três.
>> > >> De fato, seria razoável esperar que P estivesse mais próximo do maior 
>> > >> lado e conjecturar, por exemplo, que o P que minimiza a expressão é tal 
>> > >> que a/h_a = b/h_b = c/h_c.
>> > >> O fato de P ser o incentro não me parece a conjectura mais evidente 
>> > >> neste caso.
>> > >>
>> > >>
>> > >> On Sun, Aug 16, 2020 at 

[obm-l] Re: [obm-l] Re: [obm-l] Re: [obm-l] Re: [obm-l] Geometria plana com desigualdade de médias?

2020-08-20 Por tôpico Anderson Torres
Em qui., 20 de ago. de 2020 às 22:03, Anderson Torres
 escreveu:
>
> Em ter., 18 de ago. de 2020 às 19:51, Pedro José  
> escreveu:
> >
> > Boa noite!
> > Cláudio,
> > não consegui nada geométrico.
> > O máximo que atingi foi:
> > a/ha + b/hb + c/hc= [cotg(A1) +cotg (A2)]  + [cotg(B1) +cotg (B2)] + 
> > co[tg(C1) +cotg (C2)] com A1 + A2 = A; B1 + B2 + B e C1 + C2 = C.
> > Para ser mínimo cada termo entre colchetes deve ser mínimo, o que ocorre 
> > quando A1 = A2; B1 = B2 e C1 = C2. Logo P seria o encontro das bissetrizes 
> > e logo I.
> > Onde: A1= PAB e A2=PAC; B1=PBA e B2=PBC; C1=PCA e C2=PCB.
>
> Acho que daqui poderia sair uma interpretação mais escamoteada.
> Afinal, trigonometria é uma espécie de "ponto de contato" entre a
> geometria analítica e a sintética, entre a nuvem de desenhos e a de
> números.
>
> Acredito que a solução aqui seria arranjar uma interpretação
> geométrica desses colchetes de co-tangentes. Acredito que possamos
> apelar para Ptolomeu em algum momento ou para um macete de
> semelhanças, pois as projeções de um ponto sobre duas retas criam um
> quadrilátero cíclico.

Acrescentando mais coisas: se queremos minimizar cot(x) +cot(y) com
x+y fixo, isto é equivalente a minimizar tan(90-x)+tan(90-y) com
90-x+90-y fixo. Ou como maximizar tan(x) + tan(y) com x+y fixo.

Geometricamente, tangente é cateto oposto dividido por cateto
adjacente. Logo uma soma de tangentes com catetos adjacentes iguais
equivale a uma soma de catetos opostos! Assim sendo, nosso problema
pode ser pensado da seguinte forma:

Dados um ponto A e uma reta d fixos, temos que construir duas retas x
e y, com ângulo 'alfa' entre elas, ambas passando por A e tais que a
distância entre os pontos X e Y, que elas geram ao intersectar d, seja
mínima.

Daí fica fácil argumentar que a altura por A também tem que ser a
bissetriz por A.

No fundo do fundo é uma forma de geometrizar a solução trigonométrica.
A trigonometria se torna apenas um atalho.

Vou formalizar isso mais tarde, com desenhos e tudo.



>
> Isso até me lembra o famoso artigo do Shine sobre geometria cearense
> VS geometria paulista:
> https://cyshine.webs.com/geometria-2005.pdf
>
>
> >
> > Saudações,
> > PJMS
> >
> > Em ter., 18 de ago. de 2020 às 11:34, Claudio Buffara 
> >  escreveu:
> >>
> >> Será que tem uma demonstração mais geométrica e menos algébrica disso? E 
> >> que torne o resultado mais intuitivo?
> >> É razoável que o ponto P não esteja muito próximo de qualquer dos lados, 
> >> pois neste caso, se P se aproximasse do lado a, por exemplo, a/h_a 
> >> cresceria e a expressão se afastaria do valor mínimo.
> >> Mas, com lados não necessariamente congruentes, não é óbvio, a priori, que 
> >> P deva ser equidistante dos três.
> >> De fato, seria razoável esperar que P estivesse mais próximo do maior lado 
> >> e conjecturar, por exemplo, que o P que minimiza a expressão é tal que 
> >> a/h_a = b/h_b = c/h_c.
> >> O fato de P ser o incentro não me parece a conjectura mais evidente neste 
> >> caso.
> >>
> >>
> >> On Sun, Aug 16, 2020 at 10:11 AM Matheus Secco  
> >> wrote:
> >>>
> >>> Olá, Vanderlei.
> >>> Por Cauchy-Schwarz, temos
> >>>
> >>> (a/ha + b/hb + c/hc) * (a*ha + b*hb + c*hc) >= (a+b+c)^2.  (#)
> >>>
> >>> Como (a*ha + b*hb + c*hc) = 2S, onde S é a área de ABC, segue que a 
> >>> expressão a/ha + b/hb + c/hc é pelo menos 2p^2/S, onde p é o 
> >>> semi-perimetro.
> >>>
> >>> Por outro lado, a igualdade em (#) ocorre se, e somente se, ha = hb = hc, 
> >>> ou seja, quando P é o incentro do triângulo
> >>>
> >>> Abraços,
> >>> Matheus
> >>>
> >>> Em dom, 16 de ago de 2020 08:59, Professor Vanderlei Nemitz 
> >>>  escreveu:
> >>>>
> >>>> Bom dia!
> >>>>
> >>>> Tentei utilizar alguma desigualdade de médias aqui, mas não tive êxito. 
> >>>> Alguém ajuda?
> >>>> Muito agradecido!
> >>>>
> >>>> Seja P um ponto no interior de um triângulo e sejam ha, hb e hc as 
> >>>> distâncias de P aos lados a, b e c, respectivamente. Mostre que o valor 
> >>>> mínimo de (a/ha) + (b/hb) + (c/hc) ocorre quando P é o incentivo do 
> >>>> triângulo ABC.
> >>>>
> >>>> --
> >>>> Esta mensagem foi verificada pelo sistema de antivírus e
> >>>> acredita-se estar livre de perigo.
> >>>
> >>>
> >>> --
> >>> Esta mensagem foi verificada pelo sistema de antivírus e
> >>> acredita-se estar livre de perigo.
> >>
> >>
> >> --
> >> Esta mensagem foi verificada pelo sistema de antivírus e
> >> acredita-se estar livre de perigo.
> >
> >
> > --
> > Esta mensagem foi verificada pelo sistema de antivírus e
> > acredita-se estar livre de perigo.

-- 
Esta mensagem foi verificada pelo sistema de antiv�rus e
 acredita-se estar livre de perigo.


=
Instru��es para entrar na lista, sair da lista e usar a lista em
http://www.mat.puc-rio.br/~obmlistas/obm-l.html
=


[obm-l] Re: [obm-l] Re: [obm-l] Re: [obm-l] Re: [obm-l] Geometria plana com desigualdade de médias?

2020-08-20 Por tôpico Anderson Torres
Em ter., 18 de ago. de 2020 às 19:51, Pedro José  escreveu:
>
> Boa noite!
> Cláudio,
> não consegui nada geométrico.
> O máximo que atingi foi:
> a/ha + b/hb + c/hc= [cotg(A1) +cotg (A2)]  + [cotg(B1) +cotg (B2)] + 
> co[tg(C1) +cotg (C2)] com A1 + A2 = A; B1 + B2 + B e C1 + C2 = C.
> Para ser mínimo cada termo entre colchetes deve ser mínimo, o que ocorre 
> quando A1 = A2; B1 = B2 e C1 = C2. Logo P seria o encontro das bissetrizes e 
> logo I.
> Onde: A1= PAB e A2=PAC; B1=PBA e B2=PBC; C1=PCA e C2=PCB.

Acho que daqui poderia sair uma interpretação mais escamoteada.
Afinal, trigonometria é uma espécie de "ponto de contato" entre a
geometria analítica e a sintética, entre a nuvem de desenhos e a de
números.

Acredito que a solução aqui seria arranjar uma interpretação
geométrica desses colchetes de co-tangentes. Acredito que possamos
apelar para Ptolomeu em algum momento ou para um macete de
semelhanças, pois as projeções de um ponto sobre duas retas criam um
quadrilátero cíclico.

Isso até me lembra o famoso artigo do Shine sobre geometria cearense
VS geometria paulista:
https://cyshine.webs.com/geometria-2005.pdf


>
> Saudações,
> PJMS
>
> Em ter., 18 de ago. de 2020 às 11:34, Claudio Buffara 
>  escreveu:
>>
>> Será que tem uma demonstração mais geométrica e menos algébrica disso? E que 
>> torne o resultado mais intuitivo?
>> É razoável que o ponto P não esteja muito próximo de qualquer dos lados, 
>> pois neste caso, se P se aproximasse do lado a, por exemplo, a/h_a cresceria 
>> e a expressão se afastaria do valor mínimo.
>> Mas, com lados não necessariamente congruentes, não é óbvio, a priori, que P 
>> deva ser equidistante dos três.
>> De fato, seria razoável esperar que P estivesse mais próximo do maior lado e 
>> conjecturar, por exemplo, que o P que minimiza a expressão é tal que a/h_a = 
>> b/h_b = c/h_c.
>> O fato de P ser o incentro não me parece a conjectura mais evidente neste 
>> caso.
>>
>>
>> On Sun, Aug 16, 2020 at 10:11 AM Matheus Secco  
>> wrote:
>>>
>>> Olá, Vanderlei.
>>> Por Cauchy-Schwarz, temos
>>>
>>> (a/ha + b/hb + c/hc) * (a*ha + b*hb + c*hc) >= (a+b+c)^2.  (#)
>>>
>>> Como (a*ha + b*hb + c*hc) = 2S, onde S é a área de ABC, segue que a 
>>> expressão a/ha + b/hb + c/hc é pelo menos 2p^2/S, onde p é o semi-perimetro.
>>>
>>> Por outro lado, a igualdade em (#) ocorre se, e somente se, ha = hb = hc, 
>>> ou seja, quando P é o incentro do triângulo
>>>
>>> Abraços,
>>> Matheus
>>>
>>> Em dom, 16 de ago de 2020 08:59, Professor Vanderlei Nemitz 
>>>  escreveu:

 Bom dia!

 Tentei utilizar alguma desigualdade de médias aqui, mas não tive êxito. 
 Alguém ajuda?
 Muito agradecido!

 Seja P um ponto no interior de um triângulo e sejam ha, hb e hc as 
 distâncias de P aos lados a, b e c, respectivamente. Mostre que o valor 
 mínimo de (a/ha) + (b/hb) + (c/hc) ocorre quando P é o incentivo do 
 triângulo ABC.

 --
 Esta mensagem foi verificada pelo sistema de antivírus e
 acredita-se estar livre de perigo.
>>>
>>>
>>> --
>>> Esta mensagem foi verificada pelo sistema de antivírus e
>>> acredita-se estar livre de perigo.
>>
>>
>> --
>> Esta mensagem foi verificada pelo sistema de antivírus e
>> acredita-se estar livre de perigo.
>
>
> --
> Esta mensagem foi verificada pelo sistema de antivírus e
> acredita-se estar livre de perigo.

-- 
Esta mensagem foi verificada pelo sistema de antiv�rus e
 acredita-se estar livre de perigo.


=
Instru��es para entrar na lista, sair da lista e usar a lista em
http://www.mat.puc-rio.br/~obmlistas/obm-l.html
=


[obm-l] Re: [obm-l] Álgebra

2020-08-20 Por tôpico Anderson Torres
Em sáb., 15 de ago. de 2020 às 17:57, marcone augusto araújo borges
 escreveu:
>
> Determinar todos os pares ordenados (x,y) de número racionais que são as 
> soluções da equação x^2019 + y^2019 = x^2020 + y^2020
> Desde já agradeço.

Hum, estou achando isso meio confuso.

Se x e y forem iguais, temos 2x^2019=2x^2020, x=1 ou x=0. Vamos então
supor x e y diferentes.

Podemos pegar estas duas frações como (m/d)^2019 + (n/d)^2019 =
(m/d)^2020 + (n/d)^2020 em que d é o menor valor possível.

Assim, multiplica tudo por d^2020, temos d*(m^2019 + n^2019) = (m^2020
+ n^2020).

Em outras palavras, queremos encontrar todos os ternos (m,n,d) de
inteiros tais que (m^2020 + n^2020)/(m^2019 + n^2019)=d é inteiro.

Para isso, vamos tentar procurar os primos p tais que p é divisor de
(m^2019 + n^2019) e também é divisor de (m^2020 + n^2020).
Podemos supor que este primo p exista, dado que m e n são pelo menos
1, logo (m^2019 + n^2019) é pelo menos 2, logo é fatorável.

Note que se m e n forem múltiplos de p, escrevendo m=pm' e n=dn',
obtemos d=p * (m'^2020 + n'^2020)/(m'^2019 + n'^2019). Assim sendo, d
também será múltiplo de p. Assim, obtemos uma nova solução (m',n,d')

Assim, podemos de cara excluir os casos em que m e n são múltiplos de
um mesmo primo p.

Enfim.

Se p é divisor de p é divisor de (m^2019 + n^2019) e também é divisor
de (m^2020 + n^2020), então também é divisor de (m+n)(m^2019 +
n^2019)-(m^2020 + n^2020) = m*n^2019+n*m^2019=mn(m^2018+n^2018).

Temos então três hipóteses, que não são necessariamente mutuamente exclusivas:

p é divisor de m
p é divisor de n
p é divisor de (m^2018+n^2018)

Se p é divisor de m, então também é divisor de n, e vice-versa, dado
que p é divisor da soma dos dois. Assim sendo, podemos de cara
eliminar os dois primeiros casos.

Logo, p é divisor de (m^2018+n^2018).

Acredito que, repetindo esse raciocínio mais umas 2018 vezes, chegamos
em que p deve ser divisor de m+n e de m^2+n^2.

Ou, novamente, de (m+n)(m+n)-(m^2+n^2)=2mn

Logo, p é divisor de 2. Ou seja, p é igual a 2. Ou seja, o único primo
possível nisto tudo é 2. Ou seja, (m^2+n^2)=2^k*(m+n), com m e n ambos
ímpares e primos entre si.

Assim, podemos escrever m=a+b e n=a-b para inteiros a e b. Assim,
(m^2+n^2)/(m+n)=((a+b)^2+(a-b)^2)/(a+b+a-b) = 2(a^2+b^2)/(2a)=a+b^2/a,
o que implica que a é divisor de b^2.

Mas a e b são primos entre si, pois se tivessem fatores em comum,
estes fatores apareceriam em m e n.

Logo, a=1. Mas a é maior que b, pois m=a-b é maior que 0. Logo, a=1 e
b=0. Assim sendo, m=n, e sendo primos entre si, daria m=n=1.

É isso mesmo? Só tem x=y=0 e x=y=1 como soluções?

>
> --
> Esta mensagem foi verificada pelo sistema de antivírus e
> acredita-se estar livre de perigo.

-- 
Esta mensagem foi verificada pelo sistema de antiv�rus e
 acredita-se estar livre de perigo.


=
Instru��es para entrar na lista, sair da lista e usar a lista em
http://www.mat.puc-rio.br/~obmlistas/obm-l.html
=


[obm-l] Re: [obm-l] Re: [obm-l] Re: [obm-l] Re: [obm-l] Re: [obm-l] Re: [obm-l] Re: [obm-l] polinômio irredutível

2020-08-20 Por tôpico Anderson Torres
Em seg., 17 de ago. de 2020 às 12:14, Claudio Buffara
 escreveu:
>
> Eu acho que o Eisenstein inventou este critério pra polinômios da forma x^n + 
> a ou, mais geralmente, pra polinômios ciclotômicos.
> Daí funciona bem.
>
> On Mon, Aug 17, 2020 at 11:02 AM Esdras Muniz  
> wrote:
>>
>> E se p=3, e p divide N^2+9, então p^2 divide N^2+9.
>>
>> Então o critério de Eisenstein realmente não é tão abrangente. Será que tem 
>> algum outro critério que cubra casos em que o de Eisenstein não cubra?
>>
>> Em seg, 17 de ago de 2020 09:46, Claudio Buffara  
>> escreveu:
>>>
>>> Boa! Se p <> 3 mas p divide 3N e 3N^2, então p divide N ==> p não divide 
>>> N^3 + 9.
>>>
>>> On Sun, Aug 16, 2020 at 10:51 PM Esdras Muniz  
>>> wrote:

 Tenta com x^3+9.

 Em dom, 16 de ago de 2020 15:24, Claudio Buffara 
  escreveu:
>
> f(x) em Z[x], bem entendido...
>
>
> On Sun, Aug 16, 2020 at 3:08 PM Claudio Buffara 
>  wrote:
>>
>> Que tal essa aqui?
>> Prove ou disprove que, dado um polinômio f(x), irredutível sobre Q, 
>> existe um inteiro N tal que a irredutibilidade de f pode ser provada 
>> pelo critério de Eisenstein aplicado a f(x+N).

Isso me parece uma daquelas questões ultra capciosas sobre "prove ou
disprove que existe um algoritmo que..."

Inclusive imagino que esta seja uma questão indecidível neste caso particular...

>>
>> On Sun, Aug 16, 2020 at 2:31 PM Matheus Secco  
>> wrote:
>>>
>>> O melhor jeito é pensar na contrapositiva (supondo que você esteja 
>>> falando sobre irredutibilidade em Z[x] ou até em Q[x]): se f(x) fatora 
>>> como g(x)*h(x), então f(x+a) fatora como g(x+a) *h(x+a) e é claro que 
>>> uma vez que g(x) e h(x) têm coeficientes inteiros, então g(x+a) e 
>>> h(x+a) também têm. A recíproca é essencialmente idêntica.
>>>
>>> Abraços
>>>
>>> Em dom, 16 de ago de 2020 14:11, Luís Lopes  
>>> escreveu:

 Sauda,c~oes,

 Como provar que um polinômio f(x) tendo como coeficientes números 
 inteiros
 é irredutível se e somente se f(x+a) é irredutível para algum  
 inteiro ?

 Luís




 --
 Esta mensagem foi verificada pelo sistema de antivírus e
 acredita-se estar livre de perigo.
>>>
>>>
>>> --
>>> Esta mensagem foi verificada pelo sistema de antivírus e
>>> acredita-se estar livre de perigo.
>
>
> --
> Esta mensagem foi verificada pelo sistema de antivírus e
> acredita-se estar livre de perigo.


 --
 Esta mensagem foi verificada pelo sistema de antivírus e
 acredita-se estar livre de perigo.
>>>
>>>
>>> --
>>> Esta mensagem foi verificada pelo sistema de antivírus e
>>> acredita-se estar livre de perigo.
>>
>>
>> --
>> Esta mensagem foi verificada pelo sistema de antivírus e
>> acredita-se estar livre de perigo.
>
>
> --
> Esta mensagem foi verificada pelo sistema de antivírus e
> acredita-se estar livre de perigo.

-- 
Esta mensagem foi verificada pelo sistema de antiv�rus e
 acredita-se estar livre de perigo.


=
Instru��es para entrar na lista, sair da lista e usar a lista em
http://www.mat.puc-rio.br/~obmlistas/obm-l.html
=


[obm-l] Re: [obm-l] Re: [obm-l] Álgebra

2020-08-05 Por tôpico Anderson Torres
Em ter., 14 de jul. de 2020 às 23:39, Pacini Bores
 escreveu:
>
> A expressão pedida ao quadrado é igual a 4, sem usar complexos.
>
> Pacini
>
> Em 14/07/2020 21:50, marcone augusto araújo borges escreveu:
>
> Se x^2 +xy + y^2  = 0, com x,y <>0
> Determinar (x/(x+y))^2019 + (y/(x+y))^2019, sem usar números complexos.

Bem, é meio óbvio que x!=y e x!=-y, senão daria 0.

Podemos supor sem perda de generalidade que x+y=1 (basta dividir x e y
pela soma)

Assim, temos x+y=1 e x^2+2xy+y^2=1, portanto xy=1.

Assim x e y são zeros do polinômio P(x)=x^2-x+1, e x^2019+y^2019 seria
calculável mediante uma recorrência.


>
> --
> Esta mensagem foi verificada pelo sistema de antivírus e
> acredita-se estar livre de perigo.
>
>
>
> --
> Esta mensagem foi verificada pelo sistema de antivírus e
> acredita-se estar livre de perigo.

-- 
Esta mensagem foi verificada pelo sistema de antiv�rus e
 acredita-se estar livre de perigo.


=
Instru��es para entrar na lista, sair da lista e usar a lista em
http://www.mat.puc-rio.br/~obmlistas/obm-l.html
=


[obm-l] Re: [obm-l] Congruência

2020-07-08 Por tôpico Anderson Torres
Em sáb., 4 de jul. de 2020 às 20:29, marcone augusto araújo borges
 escreveu:
>
> Determinar os inteiros positivos x tais que (x^5+5x2+x+1) é múltiplo de 121

Tente ver primeiro por 11. Isso já dá uma reduzida.

> --
> Esta mensagem foi verificada pelo sistema de antivírus e
> acredita-se estar livre de perigo.

-- 
Esta mensagem foi verificada pelo sistema de antiv�rus e
 acredita-se estar livre de perigo.


=
Instru��es para entrar na lista, sair da lista e usar a lista em
http://www.mat.puc-rio.br/~obmlistas/obm-l.html
=


  1   2   3   >